You are on page 1of 92

Dreptul de copyright:

Cartea downloadat de pe site-ul www.mateinfo.ro nu poate fi publicat pe un alt site i nu poate fi folosit n
scopuri comerciale fr specificarea sursei i acordul autorului

Neculai STANCIU

ARTICOLE I NOTE
DE
MATEMATIC
GIMNAZIU
&
LICEU

Buzu, 2009

Neculai Stanciu, Berca, Buzu

Dedic aceast carte soiei mele Roxana Mihaela Stanciu i copiilor


notriBogdan Andrei i Maria.

Refereni tiinifici:
Prof. gr. I Constantin Apostol, Colegiul Naional Alexandru Vlahu, Rmnicu Srat
Prof. gr. I Gheorghe Ghi, Colegiul Naional Mihai Eminescu, Buzu
Redactor: Roxana Mihaela Stanciu
Tehnoredactare computerizat: Roxana Mihaela Stanciu

Articole i note matematice

PREFA
Articolele
ce
urmeaz
au
fost
publicate
n
reviste
de
specialitate(GMB,GMA,RMT,RIM,SM, Rec.Mat, SM, etc) sub semntura prof.
Neculai Stanciu.

Buzu, 2009

Autorul

Motto:Aritmetica i Geometria dispun de resurse bogate de dezvoltare a capacitii


copilului de a se mira, de a se ntreba, de a imagina rspunsuri, de a tatona diferite ci
de rezolvare, de a stabili puni de legtur cu nelegerea naturii, a limbajului, a istoriei
i geografiei.Dar totul trebuie s se bazeze pe dezvoltarea propriei sale curioziti, n
aa fel nct el s accepte ca unic rsplat bucuria, plcerea de a nelege, prin pai
mruni, cte ceva din lumea care l nconjoar i de a se nelege pe sine.La ntrebarea
pe care o auzim mereu, din partea unor elevi, dar i din partea unor prini sau
educatori:De ce matematic pentru copii care nu-i propun s devin matematicieni? le
rspundem: Pentru c matematica este un mod de gndire cu valoare universal i
pentru c ea prilejuiete bucurii spirituale la care orice fiin uman ar trebui s aib
acces.n msura n care adolescenii vor nva s se, bucure de frumuseile
matematicii, ale tiinei, ale artei i literaturii i vor simi nevoia de a le frecventa, ei nu
vor mai suferi de plictiseal iar tentaia unor activiti derizorii, uneori antisociale, va
scdea
Savantul Academician, Solomon Marcus

Articole i note matematice


I. ISTORICUL NOIUNILOR MATEMATICE STUDIATE N GIMNAZIU I
LICEU

aria triunghiului, paralelogramului i trapezului; volumul prismei,


piramidei i trunchiului de piramid; ptrate i triunghiuri echilaterale nscrise n cerc
papirusurile egiptene i crmizile caldeene 2000 . e. n.;

egalitatea i asemnarea triunghiurilor Tales sec. VI . e. n.;

teorema catetei i nlimii, suma unghiurilor unui triunghi, numere prime,


numere perfecte, numere prietene, media aritmetic, geometric i armonic Pitagora
sec. VI . e. n.;

teorema cosinusului, teorema lui Pitagora generalizat, raionamentul


deductiv, construcii cu compasul, lunulele lui Ipocrat Ipocrat sec. IV . e. n.;

metoda exhaustiv pentru demonstrarea formulei ariei cercului i a


volumului piramidei Eudoxiu sec. IV . e. n.;

hiperbola i parabola Menecmus sec. IV . e. n.;

teorema mpririi cu rest i algoritmul lui Euclid pentru aflarea c. m. m. d.


c. a dou numere ntregi, forma numerelor perfecte, exist o infinitate de numere prime,
2 este iraional primul text care sa pstrat (Elementele) Euclid sec. III . e. n.;

concurena nlimilor i medianelor unui triunghi, axioma de continuitate,


determinarea numrului cu dou zecimale exacte, determinarea ariei elipsei ( a b)
prin metode exhaustive, 1 + 3++ (2n -1) = n2; 2n + 1 = (n + 1)2 n2; 12 + 22
+..+ n2 = n (n + 1) (2n + 1) / 6 Arhimede sec. III . e. n.;

cercul lui Apoloniu Apoloniu sec. III . e. n.;

probleme izoperimetrice Zenodor sec. III . e. n.;

ciurul lui Eratostene pentru determinarea numerelor prime Eratostene


sec. III . e.n.;

simplificarea fraciilor, rdcina ptrat i cubic, progresii aritmetice i


geometrice, metoda fan cen pentru rezolvarea sistemelor de ecuaii liniare, rezolvarea
ecuaiei de gradul II Matematica n nou cri de la chinezi sec. II . e.n.;

teorema lui Menelau Menelau sec. I;

formulele s2 = p (p-a) (p-b) (p-c), p = (a + b + c) / 2; S = p r, a b c


= 4 R S Heron sec. II; (sec. I).

teoremele lui Ptolemeu i formulele: sin2 ( / 2 ) = 1 cos ( / 2), cos ( +


) = cos cos sin sin Ptolemeu sec. II;

teorema medianei, teorema celor trei perpendiculare, teorema bisectoarei


exterioare, biraportul, proprietatea comun a conicelor Papus sec. III;

introducerea operaiilor i notaiilor prescurtate pentru necunoscute


Diofant precursorul algebrei sec. III;

numerele negative marcheaz diferena dintre aritmetic i algebra


considerate pentru prima data de indieni;

teorema congruenelor i determinarea lui cu ase zecimale exacte de la


chinezi sec. III;

algebra i trigonometria create de arabi;

regulile de calcul cu numere negative de la chinezi;

regula de trecere a termenilor dintr-o parte n alta, procedeu numit al


Djabr, de la care a venit si numele disciplinei algebra AL Horezmi sec. IX;

Cn0 + C 1n +..+C nn =2 n de la indieni;


C nm = C nm11 + C nm 1 de la arabi;

criteriile de divizibilitate cu 2, 3, 5, 9; adunarea fraciilor prin aducerea la


c. m. m. m. c.; legea creterii organice sau irul lui Fibonacci Leonardo da Pisa
(Fibonacci 1175-1240) sec. XIII;

Articole i note matematice

simbolurile +, ,
, =, x, >, <,: sfritul sec. XV;
forma actual a cifrelor sec. XV, XVI;
cifra zero sec. XVII;
rezolvarea ecuaiilor de gradul III prin radicali Cardano (1501 1576)

rezolvarea ecuaiei de gradul IV prin radicali Ferrari (1522 1565)

1545;
1545;
inventarea logaritmilor Neper (1550 1617) 1614;

teorema lui Desargues Desargues (1593 1662) 1636;

marea teorem Fermat (Conjectura lui Fermat):ecuaia xn + yn = zn, n > 2,


n N, nu are soluie n Z Pierre Fermat (1601 1665) 1637;

crearea geometriei analitice Ren Descartes (1596 1650) i Pierre


Fermat 1637;

triunghiul lui Pascal i teorema lui Pascal pentru hexagon Blaise Pascal
(1623 1662) 1640;

noiunea de probabilitate Blaise Pascal i Pierre Fermat;

creatorul probabiliti Jacob Bernoulli (1654 1705);

creatorii calculului diferenial i integral Isaac Newton (1642 1727) i


Gottfried Wilhelm Leibniz (1646 1716). Newton a elaborat metodele sale din 1665
dar nu le-a publicat. Leibniz a publicat descoperirile sale n analiza n 1675.

demonstrarea teoremei mici a lui Fermat (p > 0, prim, a Z,


(a, p) = 1
p-1
=> a 1 (mod p)); notaiile dx i ; denumirile de derivat i diferenial precum i
formulele pentru (u/v), (uv), (vu)(n), ba udv; denumirile de abscis, ordonat i
coordonat Leibniz;

teorema lui Ceva Ceva Giovani (1648 1734) 1678;

daca N = a b..e, atunci numrul divizorilor este ( + 1) ( +1).( +


1) i suma lor (1 + a + a2 ++ a) (1 + b + b2 + b) (1 + e + e2 + e) Johann
Wallis (1616 1703);

simbolul John Wallis;

regula lim f (x) / g (x) = lim f(x) / g (x) (pentru x a) a fost dat de
Johann Bernoulli, dar publicat de L Hospital (1661 1704) n 1696;

formula lui Taylor Taylor (1685 1731) 1712;

introducerea numrului e Daniel Bernoulli (1700 1782);

teorema lui Stewart 1735;

notaiile , e, i, f (x); calculul lui e cu 23 de zecimale exacte i calculul lui


cu 100 de zecimale exacte; lim (1 + x / n)n = ex (pentru n ) (1743); lista complet a
derivatelor cu demonstrarea acestora, i extinderea regulilor lui LHospital la formele
nedeterminate / , 0 i (1755); generalizarea teoremei mici a lui Fermat (n
2, n N, a Z, (a, n)= 1 => a(n) 1 mod n)) 1758; relaia v + f = m +2 pentru poliedru
convex (1750) Leonhard Euler (1707 1783);

media aritmetic media geometric media armonic Colin MacLaurin


(1698 1746) 1748;

regula lui Cramer Gabriel Cramer (1700 1782) 1750;

notaia a + b i pentru numere complexe i teorema fundamental a algebrei


Jean DAlembert (1717 1783) sec. XVIII;

este iraional Heinrich Lambert (1728 1777) 1767;

notaiile f(x), f(n) (x), Joseph Louis Lagrange (1736 1813)


1772;

introducerea simbolului [ ], pentru partea ntreaga Arien Marie Legendre


(1752 1833) 1798;

introducerea numerelor transcendente Joseph Liouville (1809 1882);

Articole i note matematice


denumirea de determinant (1801); denumirea de numr complex i
reprezentarea n plan a numerelor complexe (1832); rezolvarea problemei construirii
poligoanelor regulate (1801); 1 + 2 +..+ n = n (n + 1) / 2; notaia (n) pentru
indicatorul lui Euler; inelul Z [ i ]; demonstrarea teoremei fundamentale a algebrei Carl
Friedrich Gauss (1777 1855);

noiunile de limit, convergen, convergena seriilor i continuitate aa cum


sunt prezentate astzi; regula lui LHospital pentru 0o, o i 1 ; denumirile de linii,
coloane, ordine, elemente, diagonala principal i secundar pentru determinani (1815);
creatorul teoriei grupurilor (1815) Augustin Louis Cauchy (1789 1857);

notaia ba f (x) dx Joseph Fourier (1768 1830) 1822;

notaia de funcie de astzi i notaiile f (a + 0), f (a 0) Peter Dirichlet


(1805 1859) 1828;

denumirea de grup Evariste Galois (1811 1832) 1830;

noiunile de margine inferioar i superioar ale unei funcii, convergen


uniform Weierstrass (1815 1897) 1841;

spaiul cu n dimensiuni Arthur Cayley i Hermann Grasmann 1843;

studiul algebrelor (1843) i grupurilor (1854) noiunea de matrice

Arthur Cayley (1821 1895);

integrala Riemann ba f (x) dx Bernhard Riemann (1823 1866) 1854;

spaiu vectorial, calcul vectorial, clase, operaiile de asociativitate,


comutativitate, distributivitate, simetrie, tranzitivitate William Hamilton (1805 1865)
1853;

notaia aij = det (aij) Kronecker (1823 1891) 1853;

noiunile de inel i corp algebric R. Dedekind (1831 1836)


1871;

teoria mulimilor G. Cantor (1845 1918) 1872;

introducerea numerelor raionale prin tieturi Dedekind 1872;

transcendena numrului e Charles Hermite (1822 1901) 1873;

denumirea de subgrup Sophus Lie (1842 1899) 1874;

teorema Rouche E. Rouche 1875;

transcendenta numrului Ferdinand Lindemann (1852 1939) 1882;

introducerea axiomatic a numerelor ntregi David Hilbert (1862 1943)


1900;

rezolvarea problemei paralelismului:


- geometria hiperbolic Nikolai Ivanovici Lobacevski (1792
1856)
1829;
- geometria hiperbolic Jno Bolyai (1802 1860) 1831;
- geometria eliptic Riemann Benhard (1826 1866) 1854 ;
- geometria neeuclidic este geometria proiectiv care las o cuadric fix
Cayley Arthur (1821 1895) 1859;
- orice grup de transformri genereaz o geometrie (axiom) programul de la
Erlangen (1872) Felix Klein (1849 1925);
- sistemul axiomatic al lui Hilbert David Hilbert (1862 1943) Bazele
geometriei 1899;
Prin profunzimea ideilor i a modului de exprimare, Bazele geometriei lui
Hilbert a devenit cartea de temelie a matematicilor moderne i metoda axiomatizrii n
sensul Hilbert a fost generalizat pentru toate ramurile noi ale matematicii. Totui, pentru
uurarea nelegerii geometriei afine i euclidiene, astzi se adopt o construcie a
geometriei cu ajutorul unei axiomatizri bazate pe algebra liniar. Acest fapt este n
concordan cu schimbrile determinate de noul curriculum, de noul sistem de evaluare i
de noile manuale.

Articole i note matematice


Bibliografie:
1. N. Mihileanu Istoria matematicii, vol. 1, Editura Enciclopedic Romn,
Bucureti, 1974.
2. N. Mihileanu Istoria matematicii, vol. 2, Editura tiinific i
Enciclopedic, Bucureti, 1981.
3. N.Stanciu, Matematic gimnaziu & liceu, Editura Rafet, Rm. Srat, 2007

III.1. Inegalitatea izoperimetric


Teorema ce urmeaz i propune s dea rspuns la urmtoarea ntrebare:
Dintre toate curbele plane regulate, simple i nchise, avnd aceeai lungime L, care
mrginete domeniul cu aria maxim?.
R = multimea numerelor reale, R 2 = {( x1 , x 2 ) / x1 R, x 2 R} =

spatiu vectorial /R.


x, y = produsul scalar a doi vectori din R 2 .

E 2 = ( R 2 , .,. ) = spatiul vectorial euclidian.


Def 1. Fie I R, interval.Se numeste curb n spatiul E 2 ,
o aplicatie C - diferentiabil
c : I E 2 , t c(t) = (x(t), y(t)) E 2 , t I .
Def 2. O curb C : [a , b] E 2 se numeste curb regulat dac
c(t) 0 t [a, b].

Articole i note matematice


Def 3. O curb C : [a , b] E 2 se numeste curb simpl dac t 1 , t 2 [a , b )
cu t 1 t 2 , avem c(t 1 ) c( t 2 ).

Def 4.O curb C : [a , b ] E 2 se numeste curb inchis dac


c(a) = c(b) si c (i) (a ) = c ( i ) (b) i  0.

Def 5. O curb C : I E 2 este parametrizat canonic, dac


c(s) = 1, s I .

Pentru a stabili prima egalitate din (1) vom folosi formula lui Green
Q P
Imc P(x, y)dx + Q(x, y)dy = D ( x y )dxdy, unde P si Q sunt
functii diferentiabile definite pe D.
Daca in formula lui Green luom Q(x, y) = x si P(x, y) = -y atunci obtinem :
S = dxdy =
D

1
[x( s) y (s) y( s) x ( s)]ds = x( s) y (s)ds = x ( s) y(s)ds.
2 a
a
a

Teorema : Consideram o curba plana regulata, simpla si inchisa, avand lungimea


L si fie S aria domeniului
D marginit de curba C.Atunci () 4 L2 .
Semnul egal are loc daca si numai daca curba C este un cerc.
Inegalitatea () poarta numele de inegalitatea izoperimetrica.

Demonstraie:
Fie t1 i t2 dou drepte paralele, tangente la curba dat astfel nct toate punctele curbei
s se gseasc n regiunea cuprins ntre t1 i t2 .Fie 2r distana ntre cele dou drepte i
C(O,r) un cerc tangent dreptelor t1 i t2.
Alegem sistemul de axe carteziene ortogonale cu originea n O i ax a absciselor
paralela la t1 dus prin O.
Curba considerat este:

Articole i note matematice

C : [0, L ] E 2 , c( s) = ( x( s ), y ( s)), c ( s) = 1, s [0, L ].


Pe cercul C(O, r) alegem ca parametru pe s.
Deci cercul C(O, r) este imaginea aplicatiei diferentiabile :
C1 : [0, L ] E 2 , c1 ( s ) = ( x( s), y ( s )).
Folosind formula (1) din lema anterioara, obtinem ca aria cercului
este data de :
L

(2)r 2 = x( s ) y ( s)ds
0

Aria S a domeniului D marginit de imaginea aplicatiei C este :


L

(3)S = - x ( s) y ( s)ds,
0

Din (2) si (3) r 2 + S = [x ( s ) y ( s ) y ( s ) x ( s)]ds


0

[x( s) y ( s) y( s) x( s)]2 ds =

[x

][

( s ) + y 2 ( s ) y 2 ( s ) + x 2 ( s ) [x( s ) x ( s ) + y ( s ) y ( s )] ds =
2

= r 2 [x( s ) x ( s ) + y ( s ) y ( s )] ds r 2 ds = rL,
0

Inmultind (6) si (8) obtinem :


r 2 L2
r 2 + S
)rL r 2 S

2
4
()4S L2 (adica tocmai inegalitatea care trebuia stabilita).
Singurul lucru pe care l mai avem de artat este acela c n (*) avem egalitate dac i
numai dac curba C este un cerc.
Necesitatea:
Presupunem c curba C este un cerc de raz r, atunci avem:
4S = 4 r 2 = (2r) 2 = L2 .
Suficiena:
Reciproc s presupunem c avem egalitatea:
4S = L2
i s demonstrm c curba C este un cerc
(r 2 + S ) r 2 S (

Articole i note matematice

(9) r 2 4S = L2 r 2 , (9 ) 2 r 2 S = rL
Din (8) si (6) (10) 2 r 2 S r 2 + S rL (11) r 2 + S = rL
Tinand seama de drumul parcurs pentru stabilirea inegalitat ii (6),
egalitatea (11)
ne arata ca trebuie sa avem :
(12)x( s ) x ( s ) + y ( s ) y ( s ) = 0, s [0, L ]
(13 ) x( s ) y ( s ) y ( s ) x ( s )  0, s [0, L ]

Deoarece curba C este regulata (12) poate fi scrisa sub forma (12)

: [0, L] R este o functie diferentiabila


Din (12) 2 ( s ) =

y (s)
x( s )
=
= ( s ), unde
y( s )
x( s )

x 2 (s) y 2 (s) x 2 (s) + y 2 (s)


=
=
= r 2 (14) ( s ) = r, unde = 1 sau = -1
y 2 ( s ) x 2 ( s ) y 2 ( s ) + x 2 ( s )

x( s ) = ry( s )
Din (12) si (14) (15)
Tinand seama de (15) conditia
y ( s ) = rx( s )
(13) (ry2 (s) + rx2 (s)  0) implica = 1
y (s)
Deci avem y ( s ) = rx( s ) x( s ) =
si din x 2 ( s ) + y 2 ( s ) = 1,
r
y( s )
y 2 (s)
1
= care prin integrare
avem 2 + y2 ( s ) = 1
r
r
r 2 y 2 (s)
s
duce la (16) y ( s ) = r sin( + s0 ), s0 = cons tan ta .din (16) si
r
s
y ( s) = rx( s) x( s ) = sin( + s0 )
r
s
(17) x( s) = rcos( + s0 ) + a, a = cons tan ta.
r

Din (16) si (17) (18)[x( s ) a ] + y 2 ( s ) = r 2


2

Din 4S = L2 si (11) (19)L = 2r


Din (18) si (19) curba cautata este cercul cu centrul
in punctul (a,0) si raza r, reprezentat de curba
s
s
C : [0, L] E 2 , c(s) = ( rcos( + s 0 ) + a, rsin( + s 0 )).
r
r

Observaie:Este evident c aria domeniului plan mrginit de o curb nchis i convex


avnd lungimea L, este mai mare dect aria domeniului plan mrginit de o curb nchis
neconvex avnd lungimea L.Din aceast cauz n demonstraia teoremei curba C am
considerat-o convex.

10

Articole i note matematice


III.2. Cteva observaii ale ,, inegalitii izoperimetrice

O1. n 1827, Jacob Steiner (17961863) a demonstrat pentru prima oar teorema
urmtoare:
,,Dintre toate figurile plane, convexe, izoperimetrice (adic care au aceeai lungime)
aria maxim este realizat de cerc.
O2. n 1916, Blaschke Wilhelm (1885-1961) demonstreaz urmtoarea teorem:
,,Pentru orice curb plan, nchis, de lungime L i arie A avem 4 A L 2 .
4 A=L 2 curba este un cerc.
O3. n 1921, Carleman Torsten (1892-1949) a demonstrat inegalitatea izoperimetric
pentru curbe pe suprafee minimale.
O4. n 1933, E.F.Beckenbach i F.Rad demonstreaz inegalitatea izoperimetric
pentru curbe pe suprafee de curbur gaussian negativ.
III.3. Consecine ale inegalitii izoperimetrice:
C1. Dintre toate triunghiurile izoperimetrice (care au acelai perimetru) aria
maxim o are triunghiul echilateral (Zenodor, sec.2 .Hr.).
Demonstraie: Se va ine cont de urmtoarea propoziie:
,, Dac factorii unui produs au suma constant, atunci produsul lor este maxim
dac factorii sunt egali. Avem S = p ( p a ) ( p b) ( p c) , aria triunghiului i
( a + b + c)
a+b+c = const.(din ipotez), p =
= const; a, b, c- variabile.
2
S este maxim S 2 este maxim produsul ( p a) ( p b) ( p c) cu suma

factorilor constant este maxim p-a = p-b = p-c a = b = c (q.e.d.).


C2. Dintre toate patrulaterele inscriptibile, izoperimetrice, aria maxim o are ptratul
(Zenodor).
Demonstraie: Considerm patrulaterul inscriptibil de laturi a, b, c, d. Din ipotez,
a+b+c+d = const. Aria patrulaterului inscriptibil, este dat de formula: S =
(a + b + c + d )
( p a ) ( p b) ( p c) ( p d ) , unde p =
= const. Avem suma: (p2
a)+(p-b)+(p-c)+(p-d) constant, i din propoziia p-a = p-b = p-c = p-d a=b=c=d
(q.e.d.).
C3. Un poligon de laturi date, are aria maxim, dac este inscriptibil. (enunat de
Christian Huygens (1629-1695) n 1675 i demonstrat de Gabriel Cramer (1704-1752)
n 1752).
Demonstraie: Fie dou poligoane P i P' formate cu aceleai laturi, cu P nscris ntrun cerc i P' neinscriptibil. Pe laturile poligonului P' purtm exterior segmente de cerc,
corespunztoare laturilor poligonului P. Obinem astfel o linie curb (C') izoperimetric
cu (C). Din inegalitatea izoperimetric avem Aria (C) > Aria (C') .

11

Articole i note matematice

Aria (C) = Aria (P) + Aria (segm. de cerc) >Aria (P') + Aria (segm. de cerc)= Aria
(C')
Deci Aria (P) > Aria (P') (q.e.d.).
C4 . Dintre toate poligoanele izoperimetrice cu acelai numr de laturi, poligonul
regulat are aria maxim. (Zenodor)
Demonstraie: Din consecina 3 avem c poligonul de arie maxim este inscriptibil.
Pe de alt parte, acest poligon trebuie s aib laturile egale. n caz contrar, presupunem
AB BC i construim triunghiul isoscel AB'C cu acelai perimetru cu ABC. Dar
Aria (AB'C) > Aria (ABC) i obinem un poligon izoperimetric de arie mai mare, ceea
ce este contrar ipotezei. Poligonul care extremeaz aria, are deci toate laturile egale, i
fiind inscriptibil, este regulat.

C5. Dintre toate poligoanele echivalente (cu aceeai arie), de acelai numr de
laturi, poligonul regulat are perimetrul minim.
Demonstraie: Fie P un poligon oarecare, de arie a i perimetru , i P', P" dou
poligoane regulate de acelai numr de laturi cu P, astfel nct P' este echivalent cu P
(a'=a) i P" izoperimetric cu P ("=).
Deoarece P" este izoperimetric cu P i P" este regulat, rezult din C4 a">a.
a" > a
a">a' ">'. Din "= i ">'>'P' are perimetrul minim (q.e.d.).

a = a '
Not: Am optat pentru aceste consecine deoarece, pot fi nelese uor de elevii din
liceu i de cei din clasele terminale din gimnaziu.
Bibliografie:
1. N. Mihileanu - ,,Istoria matematicii, vol.1, Editura Enciclopedic Romn,
Bucureti, 1974.
2. N. Mihileanu - ,,Istoria matematicii, vol. 2, Editura tiinific i Enciclopedic,
Bucureti, 1981.
3. L. Nicolescu - ,,Geometrie, Editura Universitii Bucureti, 1993.
4. N.Stanciu, Matematic gimnaziu & liceu, Editura Rafet, Rm. Srat, 2007

12

Articole i note matematice


IV. Elemente de geometria triunghiului n coordonate baricentrice
(egaliti i inegaliti n triunghi).

Articolul vine uor n completarea programei colare din liceu i are scopul de a
pune n eviden noi metode de rezolvare a problemelor de geometrie i de a lrgi
orizontul matematic al elevilor.n cele ce urmeaz, voi enuna ase teoreme importante
i voi demonstra numeroase aplicaii ale acestor teoreme referitoare la unele egaliti i
inegaliti n triunghi.
Teorema
1.Se
consider
un
triunghi
fix
ABC
i
notm
BC=a,CA=b,AB=c,S=Aria(ABC).
Atunci pentru orice M E2 (unde, E2 este planul euclidian)exist i este unic tripletul
ordonat (x,y,z) R3,x+y+z=1 astfel nct
x MA + y MB + z MC = 0
i reciproc,pentru orice triplet ordonat (x,y,z) R3,x+y+z=1 exist i este unic un punct
M E2 astfel nct
x MA + y MB + z MC = 0
i n acest caz vom spune c punctul M are coordonatele baricentrice (x,y,z) n raport cu
triunghiul ABC i vom nota M(x,y,z).Pentru orice X E2 avem
x XA + y XB + z XC = XM
(demonstraie n [1]pag.66).
Exemple de coordonate baricentrice pentru cteva puncte remarcabile ntr-un triunghi:
1.1.A(1,0,0),B(0,1,0),C(0,0,1);

1 1 1
1.2.G , , centrul de greutate;
3 3 3
a b c
centrul cercului inscris C ( I , r );
1.3.I
,
,
2p 2p 2p

a
b
c

1.4.I a
,
,
2( p a ) 2( p a ) 2( p a )
centrul cercului exinscris C ( I a , ra );

p a p b p c
punctul lui Nagel;
1.5.N
,
,
p
p
p
( p b)( p c) ( p c)( p a) ( p a)( p b)

1.6.
,
,
r (4 R + r )
r (4 R + r )
r (4 R + r )
punctul lui Gergone;
1.7.H (ctgBctgc, ctgCctgA, ctgActgB ) ortocentrul ;
R 2 sin 2 A R 2 sin 2 B R 2 sin 2C
1.8.O(
,
,
)
2S
2S
2S
centrul cercului circumscris C (O, R );
a2
b2
c2
1.9.L( 2
,
,
)
a + b2 + c2 a2 + b2 + c2 a2 + b2 + c2
punctul lui Lemoine.
Teorema 2.Puterea punctuluiM(x,y,z) E2 fa de cercul C(O,R),circumscris
triunghiului ABC este dat de relaia:
pc ( M ) = ( yza 2 + zxb 2 + xyc 2 ); OM 2 = R 2 pc (M ).
13

Articole i note matematice

(demonstraie n [1] pag.68).


Aplicaii ale teoremei 2:

a2 + b2 + c2
a2 + b2 + c2
, OG 2 = R 2
si,
9
9
a 2 + b 2 + c 2 9R 2 ;
demonstraie:se aplic teorema2 i 1.2.
2.2. pc ( I ) = 2 Rr , OI 2 = R 2 2 Rr , si, R 2r ;
demonstraie:rezult imediat din teorema2 i 1.3.
2
2.3. pc ( I a ) = 2 Rra , OI a = R 2 + 2 Rra ;
demonstraie:folosim teorema2 i 1.4.
2.4. pc ( N ) = 4r ( R r ), ON = R 2r;
demonstraie:utilizm teorema 2 i 1.5.
2.1. p c (G ) =

2.5. p c ( H ) = 8 R 2 cos A cos B cos C , OH 2 = R 2 (1 8 cos A cos B cos C ),


1
si, cos A cos B cos C , iar
8
OH 2 = 9 R 2 (a 2 + b 2 + c 2 ), deoarece, OH = 3OG.

demonstraie:se aplic teorema 2 i 1.7.


2p
2.6. pc () = r ( R + r )
;
4R + r
demonstraie:se folosete teorema 2 i 1.6.
2
2
abc
abc

2
2
2.7. pc ( L) = 3 2
, OL = R 3 2
, si,
2
2
2
2
a +b +c
a +b +c
a 2 + b 2 + c 2 4 S 3;
demonstraie:rezult imediat din teorema 2 i 1.9.
Teorema 3.a)Pentru M(x,y,z) E2 exist relaia:
xMA2 + yMB 2 + zMC 2 = pc ( M )
b)Pentru orice X E2 exist relaia:
2
xXA + yXB 2 + zXC 2 = XM 2 p c ( M ), si
xXA 2 + yXB 2 + zXC 2 yza 2 + zxb 2 + xyc 2 .
(demonstraie n [1] pag.69)
Aplicaii ale teoremei 3 :

a2 + b2 + c2
3.1.M G XA + XB + XC = 3 XG +
X E 2 ;
3
a2 + b2 + c2
;
GA 2 + GB 2 + GC 2 =
3
a2 + b2 + c2
2
2
2
XA + XB + XC
X E 2 ,
3
cu egalitate pentruX G.
2

14

Articole i note matematice

3.2.M I aXA 2 + bXB 2 + cXC 2 = 2 pXI 2 + abc, X E 2 ;


aIA 2 + bIB 2 + cIC 2 = abc;
aXA 2 + bXB 2 + cXC 2 abc, X E 2 ,
cu egalitate pentruX I .

3.3.M I a bXB 2 + cXC 2 + abc = aXA 2 + 2( p a ) XI a , X E 2 ;


X I a bI a B 2 + cI a C 2 + abc = aI a A 2 ; bXB 2 + cXC 2 + abc aXA 2
cu egalitate pentruX I a .
3.4.DacaM ( x, y, z ) E 2 , atunciX E 2 avem :
yza 2 + zxb 2 + xyc 2 xXA 2 + yXB 2 + zXC 2 XM 2 + R 2 ,
cu egalitate in s tan ga, pentruX M
si egalitate in dreapta, pentruM O.
De exemplu, pentruM I
abc aXA 2 + bXB 2 + cXC 2 2 p ( XI 2 + R 2 ).

3.5.Daca, X M xMA 2 + yMB 2 + zMC 2 =


= yza 2 + zxb 2 + xyc 2 = R 2 OM 2 , si,
xMA 2 + yMB 2 + zMC 2 R 2 , cu egalitate pentruM O;
3.6.Daca, X A MA 2 = p c ( M ) + zb 2 + yc 2 , de unde, rezulta :
MA 2 + MB 2 + MC 2 = 3 p c ( M ) + ( y + z )a 2 + ( z + x)b 2 + ( x + y )c 2 .

Teorema 4.Dac M k ( xk , yk , z k ) E2 , k = 1,2 ,atunci distana ntre punctele M1,M2


este dat de relaia:
( y1 y 2 )( z1 z 2 )a 2 + ( z1 z 2 )( x1 x 2 )b 2 +
2
M 1M 2 =
.
2
+ ( x1 x 2 )( y1 y 2 )c

(demonstraie n [1] pag.70).


Aplicaii ale teoremei 4:
Utiliznd coordonatele baricentrice (vezi exemplele date)i teorema 4 obinem
urmtoarele relaii:
a2 + b2 + c2
;4.2.OH 2 = R 2 (1 8 cos A cos B cos C ) =
9
9 R 2 (a 2 + b 2 + c 2 )
4.1.OG 2 = R 2

a 2 + b 2 + c 2 = 8 R 2 (1 + cos A cos B cos C ), de unde rezulta,


a 2 + b 2 + c 2 = 8 R 2 ABCeste, dreptunghic

15

Articole i note matematice

4.3.OI 2 = R 2 2 Rr; HN = 2OI ;


4.4.NI 2 = 9GI 2 = p 2 + 5r 2 16 Rr
p 2 + 5r 2 16 Rr ;4.5.HI 2 = 4 R ( R + r ) + 3r 2 p 2 4 R( R + r ) + 3r 2 ;
p 3 2
p 3 4 R + r;
4.6.I = r 1
4 R + r

2
4S R( R + r )
4.7.LI 2 = 2
;
p r 2 4 Rr
4
4.8.G 2 =
p 2 (4 R 2 + 8 Rr 5r 2 ) r (4 R + r ) 3
2
9( 4 R + r )
2

(2 R r )(2 R + 5r ) p 2 r (4 R + r ) 3 ;
2 p 2 (2 R r )
4.9.H 2 = 4 R 2 1
2 p 2 (2 R r ) R(4 R + r ) 2 .
2
R
(
4
R
r
)
+

Teorema 5. Dac M k ( xk , yk , z k ) E2 , k = 1,2 ,atunci avem:


1
OM 1 OM 2 = R 2 ( y1 z 2 + y2 z1 )a 2 + ( z1 x2 + z 2 x1 )b 2 + ( x1 y2 + x2 y1 )c 2 .
2
(demonstraie n [1] pag.71.).
Aplicaii ale teoremei 5:
Folosind aceast teorem obinem egaliti i inegaliti importante printre care cele ce
urmeaz:

1
( y + z )a 2 + ( z + x)b 2 + ( x + y )c 2
6
1
5.1.1.OGOA = R 2 (b 2 + c 2 ); OG OA b 2 + c 2 = 6 R 2 , si, b 2 + c 2 6 R 2
6
0
m( AOG ) 90
5.1.OGOM = R 2

1
5.1.2.OGOI = R 2 ( p 2 + r 2 2 Rr ), si,2 R(3R + r ) p 2 + r 2 m( IOG ) 900 , iar ,
6
2 R(3R + r ) = p 2 + r 2 a = b = c, sau , OI OG;
p2
, si, p 3 ( R + r ) m( NOG ) 900 , iar ,
3
p = 3 ( R + r ) a = b = c, sau , ON OG;

5.1.3.OGON = ( R + r ) 2

1
5.2.OI OM = R( R + r ) ( xbc + yca + zab);
2
1
5.2.1.OI ON = R 2 + 5 Rr ( p 2 + r 2 ), si,2 R( R + 5r ) p 2 + r 2
2
0
m( ION ) 90 , iar ,
2 R( R + 5r ) = p 2 + r 2 a = b = c, sau , OI ON ;
1
5.2.2.OI OA = R 2 + Rr bc, si, aR (b + c)r m( AOI ) 90 0 , iar ,
2
aR = (b + c)r a = b = c, sau , OA OI .

16

Articole i note matematice


Teorema6.Dac

M k ( xk , yk , zk ) E2 , k = 1,2, avem :
a) IM 1 IM 2 = 4 Rr +

1
[( z1 + z2 )ab + ( x1 + x2 )bc + ( y1 + y2 )ca]
2

1
( y1 z2 + y2 z1 )a 2 + ( z1 x2 + z2 x1 )b 2 + ( x1 y2 + x2 y1 )c 2 ;
2

(demonstraie n [1],pag.73)
b)GM 1 GM 2 =

2 2
1
(a + b 2 + c 2 ) ( x1 + x2 )a 2 + ( y1 + y2 )b 2 + ( z1 + z 2 )c 2
9
6

1
( y1 z 2 + y2 z1 )a 2 + ( z1 x2 + z 2 x1 )b 2 + ( x1 y2 + x2 y1 )c 2 ;
2

(demonstraie n [1],pag.74)
1
c) MM 1 MM 2 = pc ( M ) + [ y ( z1 + z 2 ) + z ( y1 + y2 ) ( y1 z2 + y2 z1 )]a 2 + ... .
2
(demonstraie n [1],pag.75)
Observaie.Cu ajutorul acestor relaii remarcabile se pot determina ,n
particular,produse scalare,distane,egaliti i inegaliti utiliznd puncte din mulimea:
{A, B, C , O, G, H , I , I a , I b , I c , N , , L}
asociat unui triunghi ABC.(Exerciiu!).
Mai fac observaia c particularizri i unele extinderi ale coordonatelor baricentrice
sunt abordate n lucrrile [2] , [3] i articolele [4] , [5] .Un fapt care motiveaz studiul
coordonatelor baricentrice este legtura acestora cu calculul vectorial recent (relativ)
introdus n programele colare IX-XII .
Not:Problemele rezolvate aici s-au vrut ct mai elegante;ele au fost alese dintre cele
date la diferite concursuri sau publicate n diverse alte cri sau reviste.

Bibliografie
[1] .V.Nicula,Geometrie plan,Ed.Gil,2002.
[2].N.Teodorescu,.a.,Culegere
de
probleme
pentru
concursurile
de
matematic,vol.5,S.S.M.R,Bucureti,1977.
[3].M.Craioveanu,I.D.Albu,Geometrie afin i euclidian,Ed.Facla,Timioara,1982.
[4] .T. Brsan, Recreaii matematice, nr. 1 / 2002;
[5] .C. Coand, Gazeta matematic, nr. 8 / 2005;
[6].N.Stanciu, Matematic gimnaziu & liceu, Editura Rafet, Rm. Srat, 2007

17

Articole i note matematice


V. Teoremele fundamentale ale algebrei liniare, geometriei afine i euclidiene

1. Introducere.
Punctul de plecare al acestui articol l constituie un principiu emis de Felix Klein
n memoriul Consideraii comparative asupra noilor cercetri geometrice, la Erlangen,
n 1872, cunoscut sub numele de Programul de la Erlangen. Cu ajutorul acestui principiu
sunt definite: algebra liniar, geometria afin i geometria euclidian cu ajutorul
invarianilor unui grup de transformri. Prin emiterea grupului, am identificat sistemul
axiomatic ca o teorie a invarianilor fundamentali (puncte, drepte, relaia de inciden, de
ordine, de egalitate, de paralelism, de continuitate) ai unui grup de transformri. n acest
sens, ca s studiem o disciplin matematic este esenial s determinm grupul n raport
cu care noiunile ei sunt invariante.
n elaborarea acestui articol, am inut cont c acum la liceu, se adopt o
construcie a geometriei cu ajutorul unei axiomatizri bazat pe algebra liniar, care
permite mbinarea metodelor sintetic i analitic n studiul geometriei i uureaz
nelegerea geometriei afine i a geometriei euclidiene.
2. Algebra liniar
Noiunile de spaiu vectorial, aplicaie liniar precum i proprietile acestora
sunt tratate n [1] i [2]
Fie V i W dou spaii vectoriale peste corpul K cu
dim KV = n i
dim KW = m.
2.1. Teorem (fundamental a algebrei liniare).
f : V W , este aplicatie liniara daca si numai daca ecuatia ei
matriciala este de forma
y1
a11 a12 ... a1n x1

y2
a 21 a 22 ... a 2 n x 2

Y = AX ( y 3 =
), ([3], p.32)

.
.
.
. .

...

a
a
a
...
m1
m2
mn x n
y
m
2.2. Teorem. Operaia de compunere determin pe mulimea transformrilor
liniare bijective ale unui spaiu vectorial V peste corpul K o structur de grup.
Demonstratie : Fie f : V V , o transformare liniara bijectiva de ecuatie
Y = AX si g : V V
o alta transformare liniara bijectiva de ecuatie Z = BY.Avem Z = ( BA) X ,
deci (a) g D f : V V este o transformare liniara,
conform teoremei 2.1.Daca f : V V este data de ecuatia Y = AX ,
atunci X = A -1Y si conform teoremei 2.1, avem (b)f -1 : V V
este o transformare liniara .

Din (a) i (b) rezult c.c.t.d.


2.3. Definiie. Grupul din teorema 2.2 se numete grupul liniar (vectorial)
general al spaiului vectorial V i se noteaz GL(V).
2.4. Definiie. Vom numi algebra liniar a spaiului vectorial V peste corpul K,
studiul proprietile sistemelor din V care sunt pstrate de transformrile grupului
GL(V).
3.Geometria afin

18

Articole i note matematice

Fie V un spatiu vectorial peste corpul comutativ K, A o multime nevida


si f : AxA V
o aplicatie care asociaza fiecarei perechi de elemente A, B A
vectorul f(A, B) notat AB,
astfel incat : 1)A.B, C A, AB + BC = AC;2)O A, astfel
incat aplicatia f 0 : A V
f 0 (A) = OA este bijectiva.
3.1. Definiie. Aplicaia f cu proprietile de mai sus se numete structur
afin.
3.2. Definiie. Mulimea A dotat cu structura afin f se numete spaiu afin
asociat spaiului vectorial V peste corpul K. Prin convenie elementele lui A se numesc
puncte. Spaiul afin A asociat spaiului vectorial V peste corpul K cu structura afin f se
def
desemneaz deseori prin tripletul (A, V / K, f). dim K A dim K V.
Fie A1 i A2 dou spaii afine asociate spaiilor vectoriale V1 i V2 peste acelai
corp K.
3.3. Definitie. Se numeste transformare afina a spatiului afin A1 in spatiul
afin A2 o aplicatie

: A1 A2 cu proprietatea ca O A1 astfel incat aplicatia T : V1 V2 data


de T(OA 1 ) = (O) (A 1 ),
A 1 A1 sa fie liniara .Transformarea liniara T se numeste urma
transformarii afine .

3.4. Teorema (fundamental a geometriei afine) : A1 A2 este transformare


afin dac i numai dac este dat de ecuaia Y = AX +B.( dim K A1 = n, dim K A2 = m)
([3], p. 245, [4], p. 140)
3.5. Teorem. Operaia de compunere determin pe mulimea transformrilor
afine bijective ale unui spaiu afin A o structur de grup ([4], p.136).
3.6. Definiie. Grupul din teorema 3.5. se numesc grup afin i se noteaz GA
(A).
3.7. Definiie. Se numete geometrie afin studiul proprietilor invariante ale
spaiului afin la aciunea grupului afin.
4. Geometrie euclidian.
Printre spaiile afine distingem o clas important, spaiile punctuale euclidiene.
4.1. Definiie. Un spaiu vectorial real V dotat cu un produs scalar (< , >) se
numete spaiu vectorial euclidian.
4.2. Definiie. Un spaiu afin asociat unui spaiu vectorial euclidian E se
numete spaiu punctual euclidian. dim K = dim K E.
Fie E1 i E2 dou spaii vectoriale euclidiene.
4.3. Definitie. Aplicatia T : E 1 E 2 se numeste ortogonala daca
pastreaza produsul scalar.,..
Fie 1 i 2 doua spaii punctuale euclidiene asociate spaiilor vectoriale

euclidiene E1 i E2 (dim K 1 = n, dim k2 = m).

4.4. Definiie. O transformare afin : 1 2 se numete izometrie dac urma


sa T: E1 E2 este ortogonal.

19

Articole i note matematice

4.5. Teorem. T: E1 E2 este ortogonal dac i numai dac matricea asociat A


verific relaia TA A = In
m
1, j = k
(( ) aij aik = jk =
,j , k 1, n)
i =1
0 , j k
([4], p. 90).
4.6. Teorema (fundamental a geometriei euclidiene). Transformarea : 1
2 este izometrie dac i numai dac este dat de ecuaia y = AX +B i matricea A
verific relaia TAA = In .
Demonstraie: rezult imediat din definiia 4.4 i teoremele 3.4 i 4.5.
4.7. Observaii.

a)

Dac dim

1 = dim k2 = n

se obine teorema fundamental a

geometriei euclidiene n spaiul punctual euclidian n.


Teorema fundamental a geometriei euclidiene plane, respectiv teorema
b)
fundamentala a geometriei euclidiene n spaiu se obine din teorema 4.6. pentru m = n=
2, respectiv m = n = 3.
O alt demonstraie pentru teorema fundamentala a geometrie euclidiene
c)
n spaiu se bazeaz pe proprieti elementare ale izometriilor planului ([4], p. 98).
O alt demonstraie pentru teorema fundamentala a geometriei
d)
euclidiene n spaiu se bazeaz pe proprieti elementare ale izometriilor spaiului ([4],
p. 98).
4.8. Teorem. Mulimea izometriilor bijective ale unui spaiu punctual euclidian
dotat cu operaia de compunere constituie un grup.
Demonstraie. Fie : , izometrie cu urma sa T: E E. Din teorema
3.5.compunerea a dou aplicaii afine bijective este o aplicaie afin i n plus
compunerea a dou aplicaii ortogonale este o aplicaie ortogonal. Deci avem (a)
compunerea a doua izometrii bijective este o izometrie. Tot din teorema 3.5. inversa
unei aplicai afine bijective este o aplicaie afin i n plus inversa unei transformri
ortogonale este o transformare ortogonal. Deci avem (b) inversa unei izometrii
bijective este o izometrie.
Din (a) i (b) rezult c mulimea izometriilor bijective ale unui spaiu punctual
euclidian dotat cu operaia de compunere constituie un grup.
4.9. Definiie. Grupul din teorema 4.8. se numete grupul izometriilor spaiului
punctual euclidian i se noteaz GI().
4.10. Definiie. Se numete geometrie euclidian studiul proprietilor
invariante ale spaiului punctual euclidian la aciunea grupului izometriilor (studiul
acelor proprieti care sunt pstrate de transformrile grupului GI(), adic de
izometrii).
Bibliografie
[1] C. Nstsescu, C, Ni, Gh. Grigore, D. Bulacu, Matematic, Manual pentru
clasa a XII a, profil M1, E.D.P. Bucureti, 2002.
[2] M. ena, Matematic, manual pentru clasa a XII a, profil M1, Ed. Gil,
Zalu, 2002.

20

Articole i note matematice

[3] N. Soare, Curs de geometrie (Partea I), Tipografia Universitii Bucureti,


1996.
[4] A. Turtoi Geometrie, Tipografia Universitii Bucureti, 1996.
[5]N.Stanciu, Matematic gimnaziu & liceu, Editura Rafet, Rm. Srat, 2007

VI. GENERALIZAREA
INTEGRAL

UNOR

PROBLEME

DE

CALCUL

Ideea scrierii prezentului capitol mi-a fost sugerat de gsirea unor metode
generale pentru soluionarea unor probleme de calcul integral ntlnite destul de des n
Gazeta Matematic seria A i B i n alte reviste de profil (R.M.T. Timioara, R.I.M.
Braov, S..M Bacu , Recreaii Matematice Iai, etc.).

VI.I. Asupra calculului integral pentru funciile pare i impare.


Propoziia 1.
Fie c (0, ) i f : (c, c) R o funcie continu. Atunci :
b

(1) f ( x)dx =

f ( x)dx,

a, b (c, c) ;

particular,

f ( x)dx = f ( x)dx,

a (c, c) ;
a

(2) f este par dac i numai dac

f ( x)dx =

f ( x)dx,

a (0, c) ( respectiv

a (c, c)) ;
a

f ( x)dx = 0 , a (0, c) ( respectiv a (c, c)) ;

(3) f este impar dac i numai dac

(4) dac n plus f este par atunci

f ( x)dx = 2 f ( x)dx, a (c, c) ;


0

(5) (i) dac f este par atunci

xf ( x)dx = 0 , a (c, c) ;

(ii) dac

f este impar atunci

xf ( x)dx = 2 xf ( x)dx , a (c, c) ;

(iii) dac

este arbitrar atunci

f (x

)dx = 2 f ( x 2 )dx , a (c, c) i


0

xf ( x

)dx = 0 , a (c, c) .

Demonstraie .
(1) Fie a, b (c, c) , a < b fixai; fcnd substituia x = t , obinem c.c.t.d.
f
este
par,
f ( x) = f ( x) , a (c, c)
i
deci
(2) Dac
a

f ( x)dx = f ( x)dx =
0

Reciproc s presupunem c

f ( x)dx, a (0, c)

f ( x)dx = f ( x)dx, a (0, c)


21

Articole i note matematice

Atunci

( f ( x) f ( x))dx = f ( x)dx f ( x)dx = f ( x)dx f ( x)dx = 0, a (0, c)


Rezult c f ( x) f ( x) = 0 , x (0, a) . Dac x (c,0) atunci x (0, c) i prin
urmare f ( x) f (( x)) = 0 deci f ( x) = f ( x) , x (c, c) adic f este par.
(3) Dac f este impar f ( x) + f ( x) = 0 , a (c, c) i deci a (0, c) avem
a

f ( x)dx =

Reciproc

f ( x)dx + f ( x)dx = ( f ( x) + f ( x))dx = 0,

a, b (c, c), a < b

fie

fixai

conform

ipotezei

avem

f ( x)dx = f ( x)dx = 0, dar f ( x)dx = f ( x)dx + f ( x)dx + f ( x)dx


a

dar din (1)

f ( x)dx = f ( x)dx

, rezult c

f ( x)dx = f ( x)dx

deci

( f ( x) f ( x))dx = 0 ,

de unde f ( x) + f ( x) = 0 x (c, c) prin urmare f este

impar.
(4) Dac

f ( x)dx =

este

par

f ( x)dx + f ( x)dx =
0

f ( x) = f ( x) ,

avem
0

(1)

x (c, c)

deci

f ( x)dx + f ( x)dx = 2 f ( x)dx,


0

(5) (i) Dac f este par atunci funcia x xf (x) este impar i deci

( 3)

xf ( x)dx = 0 ,

a (c, c)
(ii) analog ca n (i)
(iii) rezult imediat din (i) i (ii) innd seama de faptul c funcia x f ( x 2 )
(respectiv x xf ( x 2 ) ) este par (respectiv impar).
Propoziia 2.
O funcie f : R R continu este impar dac i numai dac x R ,
x

f (t )dt = cons tan t.

Demonstraie.
x

f fiind continu admite primitive .Fie F o primitiv a sa, rezult c

f (t )dt = C

dac i numai dac F ( x) F ( x) = C , prin derivare dac i numai dac


f ( x) + f ( x) = 0 adic f este impar.
f
este
impar
f ( x) = f ( x)
implic
Reciproc
dac
x

f (t )dt =

f (t )dt + f (t )dt =
0

(1) 0

f (t )dt f (t )dt =
0

f (t )dt

f (t )dt = 0 .

VI.II. Asupra calculului integral pentru funciile pare i impare


generalizate.
22

Articole i note matematice


Definiie.
Funcia f : [a r , a + r ] R se numete a-par dac f (a + x) = f (a x), x R cu
x r , respectiv a-impar dac f (a + x) = f (a x), x R cu x r .
Propoziia II.1.
Fie f : [x 0 r , x0 + r ] R continu cu proprietatea c af ( x0 + x) + bf ( x0 x) = c , x

cu x r , a, b R , c R , atunci:
x0 + r

2cr

f ( x)dx = a + b ,

(i)

a +b 0;

x0 r

x0 + r

cr a b
(ii) f ( x)dx = +
a
a
x0 r

x0 + r

f ( x)dx

x0

Demonstraie.
Considerm , : [ r , r ] [x 0 r , x0 + r ], (t ) = x0 + t , (t ) = x0 t
f : [a r , a + r ] R este continu putem aplica scimbarea de variabil
(r )

x0 + r

f ( x)dx = f ( x)dx = f ( (t )) (t )dt = f ( x

(i)

x0 r

(r )

+ t )dt =

( a a f (x

i cum

t ))dt =

(r )

2cr b
2cr b
2cr b
+
f ( x)dx =
f ( x0 t )dt =
+ f ( (t )) (t )dt =
a
a r
a
a ( r )
a
a r

x +r

x0 + r

2cr b 0
=
+ f ( x)dx
a
a x0 r
x0 + r

.Rezult

x +r

2cr
b 0
+
(
)
f ( x)dx =
f
x
dx

a
a x0 r
x0 r

deci

2cr

f ( x)dx = a + b .

x0 r

x0 + r

x0

x0 + r

x0

x0 r

x0 r

x0

x0 r

(0)

f ( x)dx = f ( x)dx + f ( x)dx , dar f ( x)dx = f ( x)dx = f ( x

(ii)

(r )

+ t )dt =

c b
cr b
cr b
= ( f ( x 0 t ))dt = f ( x0 t )dt = + f ( (t )) (t )dt =
a a
a a r
a a r
r
(0)

cr b 0
cr b
= +
f ( x)dx = + f ( x)dx , rezult c
a a x0 + r
a a ( r )
x0 + r

f ( x)dx =

x0 r

cr b 0
+
f ( x)dx +
a a x0+ r

Propoziia II.2.
(i)
Dac

x0 + r

f ( x)dx =

x0

cr a b
+
a
a

f : [a r , a + r ] R

x0 + r

este

a+r
2 f ( x )dx , dac f este a - par
f
(
x
)
dx
=
a

ar
0 , dac f este a - impar

a+r

23

f ( x)dx .

x0

continu

atunci:

Articole i note matematice

(ii)
Produsul (ctul) a dou funcii de a-pariti diferite este o funcie a-impar i produsul
(ctul) a dou funcii de aceeai a-paritate este o funcie a-par.
Demonstraie.
Dac f este a-par, atunci f (a + x) f (a x) = 0 ; deci n II.1. punnd a=1, b=-1,
c=0, x0=a i conform II.1.(ii) rezult c :
a+r
a+r
a+r
1 ( 1 )
f
(
x
)
dx
=

f
(
x
)
dx
=
2

a
a f ( x )dx.
1
ar
Dac f este a-impar, atunci f (a + x) + f (a x) = 0 i punnd n II.1.(ii) a=b=1, c=0,

(i)

a+r

rezult c

f ( x)dx = 0

ar

(ii)

Fie f , g a-pare adic f (a + x) = f (a x) , g (a + x) = g (a x) rezult c


( f g )(a + x) = f (a + x) g (a + x) = f (a x) g (a x)
i
analog
f
f
f
( )(a + x) = ( )(a x) rezult c f g i
sunt a-pare, analog artndug
g
g
se i restul.

Propoziia II.3.
Pentru orice funcie f : [a r , a + r ] R , exist o funcie f 1 a-par i o funcie f 2 aimpar astfel nct f ( x) = f1 ( x) + f 2 ( x), x [a r , a + r ] .
Demonstraie.
f ( x ) + f ( 2a x )
f ( x) f (2a x)
Fie
rezult
c
f1 ( x) =
,
f 2 ( x) =
2
2
f ( x) = f1 ( x) + f 2 ( x) . Cum f1 (a + x) = f 1(a x) rezult c f 1 este a-par iar, cum
f 2 (a + x) + f 2 (a x) = 0 rezult c f 2 este a-impar.

Propoziia II.4.
Dac f , g : [a r , a + r ] R sunt integrabile i f este a-par atunci:
a+r

a+r

f ( x) g ( x)dx =

ar

f ( x) ( g ( x) + g (2a x))dx.
a

Demonstraie.
Din II.3 rezult c g ( x) = g1 ( x) + g 2 ( x) , unde g1 este a-par i g 2 este a-impar deci
a+r

a+r

f ( x ) g ( x )dx =

ar
( II .2 )

= 2

a+r

f ( x )( g1 ( x) + g 2 ( x))dx =

ar
a +r

f ( x) g 1 ( x)dx +

a r
( II .3)

f ( x) g1 ( x)dx = 2

a+r

ar

a+r

f ( x)( g ( x) + g (2a x))dx .


a

Propoziia II.5.
Fie f , g : [a rf , a + r ] R integrabile i f este a-impar.Atunci :
a+r

ar

a+r

f ( x) g ( x)dx =

f ( x) ( g ( x) g (2a x))dx.
a

24

( II .2 )

f ( x) g 2 ( x)dx =

Articole i note matematice


Demonstraie.
Se demonstrez analog cu propoziia II.4 utiliznd II.2. i II.3.

Concluzie. Acest capitol propune propoziii care permit calculul unor integrale
definite ce fac obiectul unor probleme publicate n Gazeta Matematic i alte reviste de
specialitate sau n unele manuale alternative de clasa a XII-a.

n continuare propun spre rezolvare urmtoarele probleme reprezentative : pb.


14847 Gazeta Matematic, nr. 7 (1975), pb. 22377 Gazeta Matematic, nr. 5 (1991),
pb. 22750 Gazeta Matematic, nr. 1 (1993), pb. 22990 Gazeta Matematic, nr. 4
(1994), pb. 23834 Gazeta Matematic, nr. 12 (1997), pb. 24094 Gazeta Matematic, nr.
3 (1999), pb. 14847 Gazeta Matematic, nr. 7 (1975), pb. Dat n concurs Gazeta
Matematic, nr. 1 (2002), pb. 25054 Gazeta Matematic, nr. 2 (2004) i dou aplicaii
ale autorului:
Aplicaia 1.
2

S se calculeze:

sin x cos 2 x
dx
2
x)(1 + sin 2 2 x)

(1 + sin
0

Soluie.
Fie f , g : [0,2 ] R, f ( x) =

sin x
cos 2 x
, g ( x) =
.
2
1 + sin 2 x
1 + sin 2 x

Se observ c f ( x) = f ( + x) , adic este par i g ( x) = g ( + x) adic


este impar.Funcia h : [0,2 ] R , h( x) = f ( x) g ( x) este conform Propoziia
impar
II.2.
(ii)
i
conform
Propoziia
II.2.(i)
avem
2
2
sin x cos 2 x
0 h( x)dx = 0 (1 + sin 2 x)(1 + sin 2 2 x) dx = 0
Aplicaia 2.

S se calculeze:
2007

2007

x 2008
n

sh 2 k 1 x

e k =1

dx
+1

25

Articole i note matematice


n

Soluie. Notm f ( x) = x 2008 , g ( x) = sh 2 k 1 x i h( x) =

1
n

e k =1
+1
Se observ imediat c f este continu i par, g este continu i impar iar
k =1

2007

h( x) + h( x) = 1 .Rezult I =

2007

x 2008
n

sh 2 k 1 x

sh

2 k 1

2007

f ( x)h( x)dx .Se

tie c o funcie

2007

+1
e k =1
arbitrar (care nu este nici par nici impar) se poate scrie ca o sum de dou funcii una
h( x ) + h( x )
par i alta impar astfel h( x) = h1 ( x) + h2 ( x), unde h1 ( x) =
= par i
2
h( x ) h( x )
h2 ( x ) =
=impar.n continuare utilizm faptul c produsul(ctul) a dou
2
funcii de aceeai paritate este o funcie par i respectiv produsul(ctul) a dou funcii
de pariti diferite este o funcie impar i obinem :
2007
2007
2007
x 2008
I=
= f ( x)h( x)dx = f ( x)(h1 ( x) + h2 ( x))dx
n
2 k 1
x
2007 sh
2007
2007
e k =1
+1
( II .2.i ) 2007

( II .2.i )

f ( x)h1 ( x)dx =

2007
( II .2.i ) 2007

2007 2009
f ( x)dx =
.
2009

Bibliografie

[1] V. Arsinte, Probleme Elementare de Calcul Integral,Ed. Univ. Bucureti, 1995


[2] D.M. Btineu Giurgiu,.a., Analiz Matematic, Ed. Matrix Rom, Bucureti, 2004
[3] Gazeta matematic 1895 - 2007

VII. Calculul integral n cazul funciilor periodice


Propoziia 1. Fie f : R R o funcie continu. Atunci avem:
a +T

a)

f este periodic de perioad T, dac i numai dac,

f ( x)dx = c(constant)
a

26

Articole i note matematice

() a R ;
b) Urmtoarele afirmaii sunt echivalente:
(i)
Orice primitiv a lui f , este periodic de perioad T;
(ii)
f este periodic, de perioad T;
a +T

(iii)

f ( x)dx = 0 , () x R
a

Demonstraie:

a) () . Din ipotez, f ( x + T ) = f ( x), () x R. Avem:


a +T

(1)

f (t )dt =

a +T

f (t )dt + f (t )dt +

f (t )dt , ()a R;

Fcnd n ultima integral, schimbarea de variabil t=y+t, y [0, T ], obinem:


( 2)

a +T

f (t )dt = f ( y + T )dy = f ( y)dy, ()a R.


0

a +T

Din (1) i (2) rezult:

f (t )dt =

f (t )dt + f (t )dt + f (t )dt =

f (T )dt , ()a R;

x +T

(). Presupunem c

f (t )dt = c, () x R

i fie F o primitiv a lui f .

Atunci , c = F ( x + T ) F ( x), () x R i deci prin derivare obinem:


f ( x + T ) f ( x) = F ( x + T ) F ( x) = 0, () x R, de unde rezult c f este periodic
de perioad T.
b) (i) (ii) Mai nti, observm c , orice primitiv a lui f este periodic de
perioad T, dac i numai dac, exist o primitiv a lui f , periodic, de perioad T,
x

dac i numai dac, funcia F ( x) =

f (t )dt , x R, este periodic de perioad T.


a

(ii) (i).
Dac
f
este
periodic,
de
perioad
avem: ( F ( x + t ) F ( x)) = f ( x + t ) f ( x) = 0, x R, i deci, exist un c R ,

T,

astfel nct F ( x + t ) F ( x) = c, x R. Atunci c = F (0 + t ) F (0) =

f (t )dt = 0 , deci
0

F ( x + t ) = F ( x) , x R i deci F este periodic, de perioad T.


(ii) (iii). Rezult din a).
(i) (iii). Dac (i) este adevrat, atunci F este periodic, de perioad T, i avem:
x +T

f (t )dt = F ( x + t ) F ( x) = 0, x R.
x

x +T

(iii) (i ) . Din

f (t )dt = F ( x + t ) F ( x) = 0, x R
x

i deci F este periodic, de perioad T.

27

Articole i note matematice


Bibliografie

[1] . V. Arsinte, Probleme elementare de calcul integral, Editura


Universitii Bucureti, 1995.

VIII. Rezolvarea analitic i sintetic a unor probleme de geometrie n


spaiu
CINCI METODE PENTRU DETERMINAREA DISTANEI DINTRE
DOU DREPTE NECOPLANARE

28

Articole i note matematice

Capitolul de fa i propune:
Prezentarea a cinci metode pentru calculul distanei dintre dou drepte
necoplanare una sintetic , dou drept aplicaii ale produsului mixt i celelalte
dou ca aplicaii ale produsului scalar.
Scopul acestui articol este de a ngloba ntr-o schem general rezolvarea acestei
probleme(determinarea distanei dintre dou drepte necoplanare) i abordarea
dintr-o alt perspectiv a problemelor de acest gen, att din punct de vedere
metodologic ct i creativ.
Problema determinrii distanei dintre dou drepte necoplanare a mai fost tratat n
G.M. nr. 9 /2004 de ctre profesorii Valentina i Ion Cicu (o metod sintetic) i n
G.M. nr. 8 /2006 de ctre regretatul prof. dr.Florin Crjan (o metod analitic ca
aplicaie a produsului scalar).
Metoda 1.(vezi G.M. nr. 9 / 2004 ) Distana dintre dou drepte necoplanare se
poate calcula fr determinarea poziiei segmentului care definete distana,
utiliznd o formul de calcul.
Astfel , pentru calculul distanei dintre dreptele necoplanare AB i CD
(not )
6 V(ABCD)
= d(AB,CD), putem utiliza formula (1) d(AB,CD)=
,
AB CD sin(AB, CD)
unde V(ABCD) este volumul tetraedrului ABCD iar ( AB, CD) este msura
unghiului dintre dreptele AB i CD. Aceast relaie este cunoscut sub numele de
formula lui Chasles.
Metoda 2. Calculul distanei cu determinarea poziiei segmentului care o definete,
utiliznd produsul mixt .Produsul mixt al trei vectori v1 , v 2 i v3 este numrul
( not )

v1 (v 2 v3 ) = (v1 , v 2 , v3 ) care se determin calculnd determinantul format cu


coordonatele celor trei vectori scrise pe liniile determinantului.

(d p.c. )

( d1 )

v1

( P1 )

N
( P2 )

v2

(d 2 )

n figura de mai sus, dreptele necoplanare sunt (d1 ) i (d 2 ) cu vectorii directori

29

Articole i note matematice


v1 i v 2 .Dreapta perpendicular comun celor dou drepte este (d p.c ) care are ca

vector director pe v .Din d p.c d1 i d p.c d 2 , rezult v = v1 v 2 .

Dac M 1 (d1 ) i M 2 (d 2 ) atunci cele dou plane

( P1 ) i ( P2 ) au ecuaiile :

( P1 ) : (r r1 , v1 , v) = 0 , respectiv ( P2 ) : (r r2 , v 2 , v) = 0 , unde r1 (respectiv r2 ) este


vectorul de poziie al punctului M 1 ( respectiv M 2 ).Ecuaia dreptei perpendiculare
comun este dat ca intersecia celor dou plane ( P1 ) i ( P2 ) .Deci avem:
(r r1 , v1 , v) = 0
( P1 )
( d p .c . )
( d p .c . )
(r r2 , v 2 , v) = 0
( P2 )

n continuare se determin coordonatele punctului M (respectiv N) ca intersecia a


dou drepte.
(d )
(d )
{M } 1 i {N } 2
(d p.c. )
(d p.c. )
n final se determin distana dintre dreptele necoplanare (d1 ) i (d 2 ) ca distan
dintre punctele M i N; d ((d1 ), (d 2 )) = d ( M , N ) .
Metoda 3. Calculul distanei dintre dou drepte necoplanare fr determinarea
poziiei segmentului care o definete.Aceast metod are la baz tot produsul mixt.
Dac dreptele necoplanare sunt (d1 ) i (d 2 ) cu vectorii directori v1 i v 2 se
consider dou puncte M 1 (d1 ) i M 2 (d 2 ) .Avem figura de mai jos:

M2

M 1M 2

v2
M1
v1

30

Articole i note matematice

Vectorii v1 , v 2 i M 1 M 2 determin un tetraedru a crui nlime este distana cutat (


vezi demonstrarea sintetic din G.M. nr. 9 / 2004).Se tie din interpretarea geometric a
produsului mixt c volumul paralelipipedului determinat de trei vectori este valoarea
absolut a produsului mixt .
Avem
d ((d1 ), (d 2 )) = nlimea tetraedrului format de vectorii { v1 , v 2 , M 1 M 2
Scriem volumul tetraedrului n dou moduri :

(1) Vtetraedru =

V paralelipiped
6

( not )

= h.

(v1 , v 2 , M 1 M 2 )
6

A b h v1 v 2 h
=

3
2
3
Din (1) i (2) rezult formula de calcul pentru distana cutat :
(v1 , v 2 , M 1 M 2 )
() h =
v1 v 2

(2) Vtetraedru =

Metoda 4. Fie (d1 ) , (d 2 ) dou drepte necoplanare i fie MN perpendiculara lor


comun.

Presupunem c sunt cunoscute dou puncte A (d1 ) , B (d 2 ) astfel nct este


cunoscut vectorul AB .Dac v1 este vectorul director al dreptei (d1 ) i v2 este vectorul
director

al

dreptei

(d 2 )

atunci

vectorul

MN

se

exprim

astfel:

MN = MA + AB + BN = v1 + AB + v2 unde numerele reale i sunt nc


MN v = 0
1
nedeterminate.Ele vor fi determinate din condiiile de ortogonalitate
MN v2 = 0
Acestea constituie un sistem de ecuaii liniare n necunoscutele i .Dup
determinarea lui i aflm d ((d1 ), (d 2 )) = MN = v1 + AB + v2 .
Metoda 5. ( vezi G.M. nr. 8 /2006 ) Determinarea distanei dintre dreptele necoplanare
(d1 ) i (d 2 ) cu determinarea poziiei segmentului MN care definete dreapta
perpendicular comun.Considerm aceeai figur ca mai sus.
Din M (d1 ) M ( , f1 ( ), f 2 ( )) , iar din N (d 2 ) N ( , g1 ( ), g 2 ( )) , unde
f1 , f 2 , g1 , g 2 sunt funcii liniare .Necunoscutele i se determin din condiiile de

MN v = 0
1
.Se determin coordonatele punctelor M i N iar apoi
ortogonalitate
MN v2 = 0
d ((d 1 ), (d 2 )) = d ( M , N ).
Exemplu. S se calculeze distana dintre o diagonal a cubului i o muchie ce nu o
intersecteaz.
31

Articole i note matematice

y
A

z
D

C
B

Pentru utilizarea celor cinci metode alegem reperul Axyz cu originea n vrful
A, iar B ( Ax, A ( Az, D ( Ay, A(0,0, a ), B(a,0,0), C (a, a,0) i D (0, a, a).
Soluie(metoda 1.) .

6 V ( AAD B)
.Dac
nlocuim
n
aceast
relaie
AA BD sin ( AA, BD )
AA = a, BD = a 3 , sin ( AA, BD ) = sin ( DD , BD ) =
a3

i V ( AA D B) =
obinem
BD a 2
6
=
=
BD a 3
a 2
.
d ( AA, BD ) =
2
Soluie(metoda 2.) .
d ( AA, BD ) = d ( M , N ); v1 = AA(0,0, a ), v 2 = BD (a, a, a ), v = v1 v 2 = ( a 2 , a 2 ,0)
x y = 0
( P1 ) : x y = 0, ( P2 ) : x y + 2 z = 0, (d p.c. ) :
x y + 2z = 0
x = 0
x + y = a
(d1 ) ( AA) :
; (d 2 ) ( BD ) :
;
y = 0
y z = 0
d ( AA, BD ) =

x = 0
x + y = a
y = 0
y z = 0

{M }
; {N }
x = y
x = y
x y + 2 z a = 0
x y + 2 z a = 0

a
a a a
a 2
M (0,0, ), N ( , , ), d ( AA, BD ) = d ( M , N ) =
.
2
2 2 2
2
Soluie(metoda 3.) .

32

Articole i note matematice


d ( AA, BD ) =

(v1 , v 2 , AB )
v1 v 2

unde

v1 = AA(0,0, a ), v 2 = BD (a, a, a ), v = v1 v 2 = ( a 2 , a 2 ,0) , AB (a,0,0) .Se obine:

a 2
.
2
Soluie(metoda 4.) . MN = v1 + v 2 + AB = (a a, a, a + a ).
(v1 , v 2 , AB ) = a 3 , v1 v 2 = a 2 2 i d ( AA, BD ) =

MN v1 = 0
+ = 0
1
1
= , =

Condiiile de ortogonalitate sunt


2
2
+ 3 = 1
MN v 2 = 0
a 2
a a
.
Avem MN ( , ,0) i d ( AA, BD ) = MN =
2
2 2

Soluie(metoda 5.) . M (0,0, ), N ( , a , a ), MN ( , a , a ) .Condiiile de


ortogonalitate
sunt
MN v1 = 0
+ = a
a
a
a a a

= = M (0,0, ), N ( , , )

2
2
2 2 2
+ 3 = 2a
MN v 2 = 0
a 2
d ( AA, BD ) = d ( M , N ) =
.
2
n ncheiere, invitm cititorii s ncerce utilizarea celor cinci metode pentru rezolvarea
unor probleme n condiii mai generale.
Bibliografie

1. G.M. nr. 9 / 2004


2. G.M. nr. 8 /2006

33

Articole i note matematice

IX. ASUPRA UNEI PROPOZIII I APLICAIILE EI

Domnului Profesor D.M. Btineu Giurgiu,


ca semn al stimei mai multor generaii
O gam destul de larg de proprieti elementare i aparent nelegate ntre ele
admite o schem comun de demonstraie.n acest articol dorim s aprofundm aceste
proprieti, relevnd mecanismul comun al demonstraiei i aducnd anumite
completri.Contextul n care ne plasm este cel din [1] i anume:
f : [a r , a + r ] R
1).
Funcia
se
numete
a-par
dac
f (a + x) = f (a x), x R

cu

x r,

respectiv

a-impar

dac

f (a + x) = f (a x), x R cu x r ;

2). Funcia f : [a b, a + b] R , continu, a, b R , b > 0 , are graficul simetric


fa de dreapta x = a dac f (a h) = f (a + h) , h [0, b] (adic f este a -par);
3).Funcia f : [a b, a + b] R , continu, a, b R , b > 0 , aregraficul simetric
fa de punctul A(a,0) dac f (a h) = f (a + h) , h [0, b] (adic f este
a impar);
4).Fie f : R R continu.Punctul C (a, b) este centrul de simetrie pentru graficul
lui f dac f (a x) + f (a + x) = 2b , x R .
Rezultatul principal al acestui articol este:
Propoziia 1.(D.M. Btineu Giurgiu) publicat n [1] i [2], fr anul apariiei

34

Articole i note matematice

Fie f : [x 0 r , x0 + r ] R continu cu proprietatea c af ( x0 + x) + bf ( x0 x) = c , x


cu x r , a, b R , c R , atunci:
x0 + r

(i)

2cr

f ( x)dx = a + b ,

a +b 0;

x0 r

x0 + r

(ii)

f ( x)dx =

x0 r

cr a b
+
a
a

x0 + r

f ( x)dx

x0

Demonstraie.
Considerm , : [ r , r ] [x 0 r , x0 + r ], (t ) = x0 + t , (t ) = x0 t
f : [a r , a + r ] R este continu putem aplica scimbarea de variabil
(r )

x0 + r

(i)

f ( x)dx =

x0 r

f ( x)dx =

(r )

f ( (t )) (t )dt =

f ( x0 + t )dt =

( a a f (x

i cum

t ))dt =

(r )

2cr b
2cr b
2cr b
f ( x0 t )dt =
+ f ( (t )) (t )dt =
=
+
f ( x)dx =
a
a ( r )
a
a r
a
a r
x +r

x0 + r

2cr b 0
=
+ f ( x)dx
a
a x0 r
x0 + r

.Rezult

x +r

b 0
2cr
f ( x)dx + f ( x)dx =

a x0 r
a
x0 r

deci

2cr

f ( x)dx = a + b .

x0 r

x0 + r

(ii)

f ( x)dx =

x0 r

x0

f ( x)dx +

x0 r

x0 + r

x0

f ( x)dx , dar

x0 r

x0

(0)

(r )

f ( x)dx =

f ( x)dx = f ( x 0 + t )dt =

c b
cr b
cr b
= ( f ( x 0 t ))dt = f ( x0 t )dt = + f ( (t )) (t )dt =
a a
a a r
a a r
r

(0)

cr b
cr b 0
= +
f
(
x
)
dx
=
+
f ( x)dx , rezult c
a a ( r )
a a x0+ r
x0 + r

cr b 0
f ( x)dx = a + a x +rf ( x)dx +
x0 r
0

x0 + r

x0 + r

x0

x0

cr a b
f ( x)dx = +
a
a

f ( x)dx .

n [3] am gsit problemele 16024, G.M 8 1976 autor Gh.Ftu i 16383, G.M 1
1977 autor Anton Reiderer care in de istoricul propoziiei de mai sus.
Aceast teorem , atribuit lui D.M. Btineu Giurgiu, a fost n anii care au
trecut redescoperit i particularizat de mai multe ori avnd pn n prezent mai multe
aplicaii.O parte din aceste aplicaii sunt prezentate n cele ce urmeaz.
Aplicaia 1.
Dac
f : [a r , a + r ] R
este
continu
atunci:
a+r
2 f ( x )dx , dac f este a - par
f
(
x
)
dx
=
a

ar
0 , dac f este a - impar

Demonstraie.
a+r

35

Articole i note matematice

Dac f este a-par, atunci f (a + x) f (a x) = 0 ; deci n Propoziia1.(ii) punnd a = 1,


b = 1 , c = 0 , x0 = a rezult c :
a+r

a+r

a+r

1 ( 1 )
f ( x )dx = 2 f ( x )dx.
ar
1
a
a
Dac f este a-impar, atunci f (a + x) + f (a x) = 0 i punnd n 1.(ii) a=b=1, c=0, rezult
f ( x )dx =

a+r

f ( x)dx = 0 .

ar

Aceast aplicaie, caz particular al teoremei D.M. Btineu Giurgiu, este foarte
cunoscut i utilizat foarte des n diverse cazuri particulare.
Aplicaia 2.
Fie f : [a b, a + b] R , continu, a, b R , b > 0 .Funcia f are graficul simetric
a+h

fa de dreapta x = a , dac i numai dac

a+h

f ( x)dx = 2

ah

f ( x)dx = 2

ah

f ( x)dx ,
a

h [ b, b] .
Demonstraie.
Dac f este simetric fa de dreapta x = a , atunci f (a h) = f (a + h) , h [0, b] ,

deci f este a - par i conform Aplicaiei 1., rezult c

a +b

a +b

a b

f ( x)dx = 2 f ( x)dx

i fcnd schimbarea de variabil ( x) = x b , : [a, a + b ] R , (a) = a b,

(a + b) = a , obinem

a +b

f ( x)dx =

f ( x)dx .

a b

Reciproc,

fiind

continu

admite

ca

primitiv

pe

F ( x) = f (t )dt ,
a

F : [a b, a + b] R , cu F (a ) = 0 . Relaia din enun, implic :

F (a + h) F (a h) = 2 [F (a ) F (a h)] = 2 [F (a + h) F (a )] , h cu h b .
Rezult c F (a + h) = F (a h) , deci F (a + h) = F (a h) (a h) , adic
f (a + h) = f (a h) i deci f este simetric fa de dreapta x = a , adic f este apar.

Aplicaia 3.
Fie f : [a b, a + b] R , continu, a, b R , b > 0 .Graficul funciei f este simetric
a +b

fa de punctul A(a,0) , dac i numai dac

f ( x)dx = 0.

a b

Demonstraie.
Dac f are graficul simetric fa de punctul A(a,0) avem f (a h) = f (a + h) ,
h [0, b] , adic f este a-impar i conform Aplicaiei 1. rezult c

a +b

f ( x)dx = 0.

a b

Reciproc, ca n demonstraia Aplicaia 2. relaia din enun devine


F (a + h) F (a h) = 0, care prin derivare devine: f (a + h) + f (a h) = 0, adic f
36

Articole i note matematice

este a - impar , deci are graficul simetric fa de punctul A(a,0).


Aplicaia 4.(Problem menionat la concursul S.S.M autor Ana Avram, G.M 1
1978, dat n concurs)
Fie f : R R continu i punctul C (a, b) centrul de simetrie pentru graficul lui f ,
2a

atunci

f (t )dt = 2ab.
0

Demonstraie.(n G.M 1 1978 avem o alt demonstraie)


Dac punctul C (a, b) este centrul de simetrie pentru graficul lui f , atunci
f (a x) + f (a + x) = 2b , x R .Conform Propoziiei 1. pentru a = b = 1 , c = 2b ,
a+a

2a

x0 = a , rezult c

f (t )dt =

f (t )dt =

aa

2 2b
a = 2ab.
1+1

Aplicaia 5.
Fie f : R R continu. C (a, b) este centrul de simetrie pentru graficul lui f dac i
a+ x

numai dac

f (t )dt = 2bx,

x R .

a x

Demonstraie.
Dac C (a, b) este centrul de simetrie pentru graficul lui f
atunci
f (a r ) + f (a + r ) = 2b , r R .Observm c putem aplica Propoziia 1. cu
a+ x

a = b = 1, c = 2b , r = x rezult c

f (t )dt =

a x

Reciproc, dac
continu
2bx =

2 2b
x = 2bx.
1+1

a+ x

a+ x

a x

a x

f (t )dt = 2bx , rezult f (t )dt + f (t )dt = 2bx

admite

a+ x

a x

primitive

fie

Fo

primitiv

dar, cum f este

sa;

atunci

avem

f (t )dt + f (t )dt = F (a) F (a x) + F (a + x) F (a) = F (a + x) F (a x) .

Derivnd
relaia
de
mai
sus
rezult
2b = F (a + x) F (a x) = f (a + x) f (a x) ( x) = f (a x) + f (a + x) c.c.t.d.
Un caz particular al acestei aplicaii este problema 17117 din G.M. 3 1978, autor
Paul Lefter.
n ncheiere, propun cititorilor s consulte [2] i [3] unde se gsesc mai multe
exerciii propuse de diveri autori , date la concursuri i olimpiade care se pot rezolva
utiliznd Propoziia 1., i proprietile funciilor pare i impare generalizate.
Indicaii:
a) din [3] : 14847 G.M. nr. 7 / 1975, 22377 G.M. nr. 5 / 1991, 22750 G. M. nr.
1 / 1993, 22990 G.M. nr. 4 / 1994, 23834 G.M. nr. 12 / 1997, 24094 G.M. nr.
37

Articole i note matematice

3 / 1999, 14847 G.M. nr. 7 /1975, pb. Dat n concurs G.M. nr. 1 /2002,
25054 G.M. nr.2 /2004, 25775 G.M. nr. 4 / 2007;
din [2]: 6.57, 6.58, 6.112, 6.117, 6.118, 6.135.

b)

Bibliografie

[1] V. Arsinte, Probleme Elementare de Calcul Integral,Ed. Univ. Bucureti, 1995


[2] D.M. Btineu Giurgiu,.a., Analiz Matematic, Ed. Matrix Rom, Bucureti, 2004
[3] Gazeta matematic 1895 2007 (ediia electronic)

X. GENERALIZAREA UNOR INEGALITI


n cinstea reunirii ntregului material din prestigiosul patrimoniu
publicat timp de 110 ani n Gazeta Matematic
(vezi Gazeta Matematic - Ediie Electronic, un produs de excepie care conine
ntreaga colecie a Gazetei Matematice seria B din perioada 1895-2005, 60 000 de
pagini ce cuprind peste 90 000 de probleme i articole matematice (echivalentul a
peste 300 de culegeri de matematic)).
n cei 113 ani de apariie nentrerupt a Gazetei Matematice (1895 2008) n
paginile acestei reviste ct i n alte reviste din ara noastr i din strintate s-au
publicat mai multe inegaliti , care permit evidenierea unei clase (de inegaliti )care
au anumite proprieti commune.
n acest articol vom exemplifica utilitatea , elegana i generalitatea folosirii conceptului
matematic de funcie convex n demonstrarea unor inegaliti.
Vom considera f : D R , o funcie convex pe mulimea D .Pentru i R +
m

cu

i =1

2
i

0 i ai D , i = 1, m avem cunoscut inegalitatea lui Jensen


m

ja j
(1) f (

j =1

j =1

j =1

f (a j )

j =1

care se poate demonstra uor prin inducie.


Dificultatea stabilirii unor inegaliti prin folosire funciilor convexe const n
alegerea funciei convexe f i a numerelor i din inegalitatea (1).
Fie i , i (0, ) ; pi , qi , k i R i funciile u i : (0, ) R date de
u i ( x) = ( i xipi + i xiqi ) ki , i = 1, n .

38

Articole i note matematice

Vom considera funcia :


n

(2) f ( x) = u i ( x)
i =1

Se arat prin inducie c :


n

f ( x) = Ai ( x)u i ( x) , unde Ai ( x) =
i =1

( x) i

j =1, j i

n
n
n

f ( x) = Ai ( x)u i ( x) + Bij ( x)u i ( x)u j ( x) , unde Bij ( x) =


i =1

i =1 j =1

( x) .

k =1, k i , j

Pe de alt parte avem :

u i ( x) = k i ( i x pi + i x qi ) ki 1 ( i pi x pi 1 + i qi x qi 1 ) , i = 1, n i

u i ( x) = k i (k i 1)( i x pi + i x qi ) ki 2 ( i pi x pi 1 + i qi x qi 1 ) 2 +
+ k i ( i x pi + i x qi ) ki 1 ( i p i ( pi 1) x pi 2 + i qi (qi 1) x qi 2 ) , i = 1, n .
Fie
D = x (0, ) /( i pi x pi 1 + i qi x qi 1 ) > 0, ( i pi ( pi 1) x pi 2 + i q i (qi 1) x qi 2 > 0, i = 1, n

atunci x D rezult c f ( x) 0 i prin urmare funcia f este convex pe D .


Din inegalitatea lui Jensen (1) se obine:
(3)

( i a
j =1 i =1

j = 1, m cu

pi
j

a
a i

+ i a ) m i ( ) pi + i ( ) qi , unde
m
m
i =1

qi
j

j =1

ki

a
j =1

= a i j = 1 ,

j = m.

Dac n inegalitatea (3) nlocuim n = 1 , atunci , (0, ) avem:


a
a
(4) (a + a ) m ( ) p + ( ) q
m
m
j =1
m

q k
j

p
j

Aplicaii (ale inegalitii (4))

n continuare, voi prezenta un set de inegaliti din Gazeta Matematic care, au


fost rezolvate la vremea respectiv prin alte metode.
1. Dac n inegalitatea (4) nlocuim = 1 , = 0 , k = 1 rezult :
m

a jp
j =1

( a j ) p
j =1

, inegalitatea Titu Andreescu, care generalizeaz problema


m
mp
8807 din G.M nr. 3 / 1968 autor Iosif Bohler i problema 8785 din G.M nr. 3 / 1968
autor N. Pantazi.
(5)

2. Dac n inegalitatea (5) p = 2 , avem :

a2
, publicat n Journal de mathematiques elementaires n 1964 i n G.M
a

m
j =1
nr. 10 / 1964 problema 6579 .
n

2
j

3. Dac n (4) nlocuim a = 1 , = = 1 i q = 1 rezult :

39

Articole i note matematice

1 k (1 + m 2 ) k
(a j + )
, problema 8745 din G.M nr. 2 / 1968, autor Liviu Pran

aj
m k 1
j =1
(care este n legtur cu problema 7877, C.d Skiliarski , 1965, pag. 67)
m

1
4. Pentru = 0 , = k = 1 , q = , s 2 , s N obinem:
s
m
m
1
, problema 8796 din G.M. nr. 3 / 1968 autor Liviu Pran ,n
ms

a
i =1 s a j
legtur cu problemele 6641 din G.M. nr. 12 / 1964 autor Cornel Popovici , 8358
din G.M. nr. 7 / 1967 autor Dan Stnescu i problema 8688 din G.M. nr. 1 / 1968.
n ncheiere, pe baza ideilor prezentate, propunem cititorilor s rezolve
urmtoarele probleme:
a) Fie a j > 0 , j = 1, m cu

a
j =1

= a . S se arate c :

a q (a r + m r )
m q + r 1
j =1
(soluia se obine imediat dac nlocuim n (4) k = 1 i p = q + r )
m

a qj (a rj + )

b) Fie a j > 0 , j = 1, m cu

a qj (a rj + 1)
j =1

a
j =1

= a . S se arate c :

a q (a r + m r )
m q + r 1

(soluia se obine imediat dac nlocuim n a) = )


c) Fie a j > 0 , j = 1, m cu
m

a
j =1

q
j

a
j =1

= a . S se arate c :

m
m
(a rj + 1)(a1 a 2 ...a m ) 1 ( ) m q r + ( ) m q
a
a

(soluie. n inegalitatea a) nlocuim = = (a1 a 2 ...a m ) 1 i inem seama c produsul a


m numere reale strict pozitive, pentru care suma este constant, este maxim atunci cnd
numerele sunt egale ntre ele, adic
a
a1 a 2 ...a m ( ) m )
m
d) Fie a j > 0 , j = 1, m . S se arate c :

40

Articole i note matematice


m

a
j =1

p
j

(a rj + 1)(a1 a 2 ...a m ) 1 m m q r + m m q ( vezi G.M nr. 10 / 1968 , problema

9234, autor Liviu Pran)


(soluie.rezult imediat dac nlocuim n d) a = 1 ).

Bibliografie

[1]. Gazeta matematic 1895 2007 (ediia electronic)


*** www.gazetamatematica.net

(1175 -1240)
Despre irul lui Fibonacci
DE NECULAI STANCIU
Abstract

The purpose of the article is to describe the contributions to Mathematics made


by the thirteenh century Italian, Fibonacci.Unfortunately, not much is known about
41

Articole i note matematice

Fibonacccis personal life.Representative problems solved by Fibonacci are set as


challenges to the reader.
After a brief historical account of Leonardo Pisano Fibonacci, some basic results
concerning the Fibonacci numbers are developed and proved, and entertaining examples
are described.Connections are made between the Fibonacci numbers and the Golden
Ratio, biological nature, and other combinatorics examples.
We are considering both the originality and power of his methods, and the
importance of his results, we are abundantly justified in ranking Leonardo of Pisa as the
greatest genius in the field of number theory who appeared between the time of
Diophantus and Fermat.
Key words: History of Mathematics, Fibonaccis Rabbits, Fibonacci numbers
and nature,Divine proportion, Golden Section in Art(Architecture, music and human
body), The Fibonacci sequence, Fibonacci identities, matrix methods.
M.S.C.: 01-XX, 01AXX, 01A05, 11B39, 11B37, 11B50.

1. Istorie.
1.1. Cine a fost Fibonacci?
Fibonacci(1175-1240) a fost unul dintre cei mai mari matematicieni ai evului
mediu.S-a nscut n Italia, n oraul Pisa, faimos pentru turnul su nclinat, care parc
st s cad.
Tatl su, Bonacci Pisano, a fost ofier vamal n oraul Bougie din Africa de
Nord , astfel c Fibonacci a crescut n mijlocul civilizaiei nord-africane.A cunoscut
astfel muli negustori arabi i indieni (deoarece a fcut multe cltorii pe coastele
Mediteranei) de unde a deprins tina lor aritmetica, precum i scrierea cifrelor arabe.
1.2. Crile lui Fibonacci.
n 1202 revine n Italia unde public un tratat de aritmetic i algebr intitulat
Incipit Liber Abacci( compositus a Leonardo filius Bonacci Pisano).n acest tratat
introduce pentru prima dat n Europa sistemul de numeraie arab, cifre pe care le
folosim i n zilele noastre:0,1, 2, 3,,9.
n 1220 public Practica Geomitriae, un compendiu de rezultate din
geometrie i trigonometrie, apoi n 1225 Liber Quadratorum n care studia calculul
radicalilor cubici.Crile lui Fibonacci au cunoscut o larg rspndire aa nct timp de
peste dou secole au fost considerate sursele cele mai competente n domeniul
numerelor.
Pentru a nelege mai bine situaia din acele vremuri trebuie s aruncm o privire
pe matematica n Europa i n Orient.
Matematic arab
Imperiul arab, odat cu apariia Islamului (sec VII), se extinde foarte repede
cuprinznd Orientul Apropiat, o parte din Asia Mic i Central, ajungnd pn la Valea
Indului, nordul Africii i Peninsula Iberic.Se ridic importante centre culturale
ca:Bagdad, Samarkand, Buhara, Horezm, Damasc, Cordoba, Granada, Sevilla, Toledo,
dup ce n prealabil fusese distruse Ispahanul, Persepolis i Alexandria.

42

Articole i note matematice

Matematica arab este matematica creat sub dominaia arab, nu neaparat


aparinnd arabilor, deoarece puini dintre matematicienii arabi erau de origine arab
dar, au asimilat foarte repede cultura Orientului precum i cea Elen pe care le transmit
n diverse pri ale imperiului.Primul mare matematician arab a fost Al-Horezmi (780
850).Din opera sa se detaeaz Algebra structurat pe 4 capitole (Soluiile
ecuaiilor,Calculul dobnzilor, Geometria, Algebra testamentar).Al-Horezmi a fost
primul matematician care a stabilit reguli pentru adunare, scdere, multiplicare i
divizare cu noile numere arabe.De la el provine cuvntul algoritm (ncercai s spunei
numele Al-Horezmi repede de cteva ori!).ntr-un alt tratat tiina transpunerii i a
reducerii, specific procesul manipulrii ecuaiilor algebrice, al-jabr, a ajuns la noi
ca algebr.
Abu Kamil (900), nscut n Egipt, este continuator a lui Al-Horezmi.n Cartea
raritilor din aritmetic se ocup cu rezolvarea n numere ntregi a sistemelor liniare
nedeterminate.
Abu Wafa (940 997) s-a ocupat cu geometria practic.n lucrarea sa Cartea
perfect expune bazele trigonometriei, inclusive teorema sinusurilor.De asemenea
rezolv probleme de trigonometrie sferic, utiliznd cu predilecie funcia cotangent.
Al-Hazem (1000) prin Cartea opticii este un precursor al acestei tiine.Tot el
formuleaz axioma lui Pasch i ncearc demonstrarea postulatului V al lui Euclid.
Omar Al-Khayyam (1048 1123), este primul matematician care expune o teorie
general a ecuaiilor de gradul III.Recent a fost descoperit un memoriu al su asupra
operei lui Euclid. Omar Al-Khayyam, conductorul Observatorului astronomic din
Ispahan, s-a ocupat i cu patrulaterul Saccheri (care, de drept ar trebui numit
patrulaterul lui Omar), apoi a dat prima formulare a axiomei lui Arhimede.Era vestit i
ca poet.
Al-Biruni (973 1048), persan de origine, este cel care n 1030 introduce cercul
trigonometric.Tot el calculeaz lungimea meridianului terestru la 41.550 km.
Nassir ed Din al Tusi (1201 1274), conductorul Observatorului astronomic
din Maraga, s-a acupat cu teoria paralelelor.n Tratatul despre patrulaterul complet a
fcut o expunere integral a rezolvrii triunghiurilor (plane i sferice).
Al-Kashi (1400), iranian de origine, n Cheia aritmeticii se ocup cu formula
binomului i cu extragerea de rdcini.S-a ocupat intens i de calcule aproximative, iar
n Tratatul despre circumferin din 1424, d valoarea numrului cu 16 zecimale
exacte.
De aici, matematica i n general cultura arab decade.
Matematica evului mediu.
Cruciadele (campanii pentru recucerirea locurilor sfinte), prilejuiesc stabilirea de
legturi cu cultura arab musulman (mai ales n Spania i Sicilia) precum i cu
Bizanul.
Dup ce Spania este recucerit de mauri, Toledo devine centru cultural de
prestigiu.n acest moment ncep traducerile din arab.Printre primii traductori este
englezul Adelard de Bath (1100) care, deghizat ca student mahomedan la Cardoba
traduce din limba arab Elementele lui Euclid i Algebra lui Al-Horezmi, iar din
limba greac, opera lui Ptolemeu.Din aceeai perioad se remarc i ali traductori ca:
Ioannes din Sevilla i Gerardo din Cremona (1114 1187) care au tradus circa 80 de
lucrri clasice din limba arab .Ambii au lucrat la Toledo.
Secolele XII XV reprezint perioada de asimilare a matematicii antice i a
celei orientale.
Leonardo da Pisa, este pe drept considerat primul mare matematician original
al Europei.n numeroasele sale cltorii (Egipt, Siria, Grecia, Sicilia) ia contact cu cu
cultura elen i cea arab .
43

Articole i note matematice


LIBER ABACI O CARTE REMARCABIL

Povestea numerelor apare n Italia n 1202, o dat cu apariia crii Liber Abaci,
scris de Leonardo Pisano, pe atunci n vrst de 27 de ani.Cartea, are 15 capitole, i
sunt scrise n ntregime de mn, tiparul aprnd 300 de ani mai trziu.Leonardo, a fost
inspirit s scrie cartea dup o vizit la Burgia, un ora prosper Algerian, unde tatl su
era consul de Pisa.n acest timp,Fibonacci a nvat secretele sistemului de numere
indo-arab, pe care arabii l-au introdus n Vest n timpul cruciadelor.
Cartea a atras numeroi adepi n rndul matematicienilor din Italia, precum i
din restul Europei.Liber Abaci, a dezvluit oamenilor o cu totul alt lume, unde
numerele au nlocuit literele.Fibonacci ncepe cartea cu noiuni despre identificarea
numerelor, de la uniti la cifra zecilor, a sutelor, a miilor etc.n ultimile capitole gsim
calcule cu numere ntregi i fracii, regulile proporiilor, extrageri de rdcini ptrate i
de ordin superior, apoi se prezint soluiile ecuaiilor liniare i ptratice
Liber Abaci era plin cu exemple practice:calcule de contabilitate financiar,
calculul profitului, schimbul de bani, conversia greutilor, calculul mprumutului cu
dobnd (interzis n acel timp n diverse locuri ale lumii).
Dei era cunoscut n anul 1000, i dei Liber Abaci a explicat avantajele ,
sistemul de numrare, indo-arab, nu a prins la scar mare pn aproape n 1500
e.n.Motivele au fost, n mare parte dou.Primul ine de ineria uman i rezistena la
schimbare a omului, pentru c nvarea unui sistem radical nou cere timp i de faptul c
biserica catolic din acea perioad considera cifrele arabe de origine pgn.Al doilea
motiv este de natur practic, deoarece era mult mai uor s se comit fraude.Era
tentant schimbarea lui 0 n 6 sau 9, iar 1 putea fi uor nlocuit cu 4, 6, 7, sau 9 (de
atunci europenii scriu 7 cu codi!).

Dei noile numere au aprut n Italia, Florena a emis un edict n 1229 prin care
interzicea bancherilor folosirea simbolurilor infidele.Ca rezultat, muli dintre cei care
voiau s nvee noul sistem se deghizau n musulmani.
Originea sistemului de numere.
Putem aprecia succesul lui Fibonacci cu Liber Abaci doar dac privim cum a
evoluat societatea, din punctul de vedere al numerelor, pn la el.Msurarea i
numrarea au aprut cu cteva zeci de mii de ani naintea lui Hristos.Oamenii au
nfiinat primele aezri pe malurile Tigrului i Eufratului, Nilului, Gangelui, Indului i
Amazonului.Fluviile erau folosite pentru comer i transport, iar aventurierii au
descoperit mrile i oceanele unde se vrsau apele.Cltoriile pe distane lungi cereau
msurarea timpului i calcule precise.Preoii erau de obicei astronomi, iar din
astronomie a venit matematica .

44

Articole i note matematice

n 450 .e.n., grecii au inventat un sistem numeric alfabetic, care folosea cele 24
de litere ale alfabetului grecesc i alte trei litere , care mai trziu au disprut.Fiecare
numr de la 1 la 9 avea propria liter, la fel i multiplii de 10.Alfa, nsemna 1, iar ro
reprezenta 100.Astfel, 112 se scria ro-deca-beta.Acest sistem se putea folosi cu
greutate pentru calcule.Abacul, era cel mai vechi aparat de numrat din istorie.
Un occidental, matematician din Alexandria, Diofantus, prin 250 e.n., a sugerat
un sistem de numere comparative cu sistemul de litere.Remarcabilele sale invenii au
fost ignorate vreme de 1500 de ani.Pn la urm, lucrarea sa a fost recunoscut cum se
cuvine i a jucat un rol important n algebra secolului al XVII-lea.Ecuaiile algebrice ,
de forma ax + by = c , se numesc ecuaii diofantice.
Piesa central a sistemului indo-arab a fost inventarea lui zero, sunya la
indui, cifr n arab, tsfira n rusete ceea ce nseamn numr.Terminul provine
de la cipher, ceea ce nseamn gol i se refer la coloana goal de la abac.
1.3. irul lui Fibonacci.Numele Fibonacci.
Fibonacci a rmas n memoria noastr prin irul : 0, 1, 1, 2, 3, introdus n
anul 1202, atunci matematicianul fiind sub numele de Leonardo Pisano (Leonard din
Pisa).
Mai trziu, matematicianul nsui i-a spus Leonardus Filius Bonacii Pisanus(Leonard
fiul lui Bonaccio Pisanul).n secolul al XIV-lea irul prezentat mai sus a fost denumit
irul lui Fibonacci prin contracia cuvintelor filius Bonacii.Acest ir apare pentru prima
dat n cartea menionat mai sus Liber Abaci(Cartea despre abac), fiind utilizat
n rezolvarea unei probleme de matematic.
1.4.Iscusina lui Fibonacci.Problema iepurilor.Originea irului Fibonacci.
Potrivit obiceiului din acea epoc, Fibonacci a participat la concursuri
matematice(adevrate dispute publice) pentru cea mai bun i mai rapid soluie a unor
probleme grele(ceva n genul Olimpiadelor Naionale).Iscusina de care ddea dovad n
rezolvarea problemelor cu numere uimise pe toat lumea, astfel c reputaia lui
Leonardo a ajuns pn la mpratul Germaniei, Frederik al II-lea.La un concurs
prezidat de acest mprat una din probleme date spre rezolvare a fost: s se gseasc un
ptrat perfect, care s rmn ptrat perfect dac este mrit sau micorat cu 5.Dup un

1681 41
=
timp scurt de gndire Fibonacci a gsit numrul
144 12
2

.ntradevr:

1681
961 31
1681
2401 49
5 =
= i
+5=
= .Nu se tie raionamentul lui
144
144 12
144
144 12
Fibonacci dar, toate ncercrile, chiar i cele mai ingenioase, de a rezolva aceast
problem cu ajutorul algebrei, duc n cel mai bun caz la o ecuaie cu 2 necunoscute.
La un alt concurs prezidat de mprat problema propus concurenilor suna
astfel:
Plecnd de la o singur pereche de iepuri i tiind c fiecare pereche de iepuri produce
n fiecare lun o nou pereche de iepuri, care devine productiv la vrsta de o lun,
calculai cte perechi de iepuri vor fi dup n luni (se consider c iepurii nu mor n
decursul respectivei perioade de n luni).
Soluie.Din datele problemei rezult c numrul perechilor de iepuri din fiecare lun
este un termen al irului lui Fibonacci.ntr-adevr, s presupunem c la 1 ianuarie exista
o singur pereche fertil de iepuri.Notm cu 1 perechea respectiv .Ea corespunde
numrului f 2 din irul lui Fibonacci:
f 2 = f 0 + f1 = 0 + 1 = 1 .
45

Articole i note matematice

La 1 februarie, mai exist o pereche pe care o notm 1.1.Deci n acest moment sunt
dou perechi , ceea ce corespunde termenului:
f 3 = f1 + f 2 =1+1=2.
La 1 martie sunt 3 perechi, dou care existau n februarie i una nou care provine de la
perechea numrul 1(se ine seama c o pereche devine fertil dup dou luni).Notm cu
1.2 aceast nou pereche.Numrul perechilor din aceast lun corespunde termenului:
f4 = f2 + f3 = 1 + 2 = 3 .
La 1 aprilie exist 5 perechi i anume:trei perechi existente n luna martie , o pereche
nou care provine de la perechea 1 i o pereche nou care provine de la perechea 1.1
care la 1 martie a devenit fertil (pereche pe care o notm cu 1.1.1).Numrul perechilor
din aceast lun corespunde termenului:
f 5 = f 3 + f 4 =2+3=5.
Termenii din aceast relaie se interpreteaz astfel:
f 4 =numrul perechilor existente n luna precedent ;
f 3 =numrul perechilor noi(provin de la perechile existente n luna anteprecedent).
Procednd n continuare n acest fel, vom deduce c la data de 1 decembrie numrul
perechilor este dat termenul:
f13 = f11 + f12 = 89 + 144 = 233 ,
iar la 1 ianuarie anul urmtor exist:
f14 = f 12 + f13 = 144 + 233 = 377 perechi de iepuri.
Concluzia este urmtoarea :
Dac notm cu f n numrul de perechi de iepuri dup n luni , numrul de perechi de
iepuri dup n + 1 luni, notat cu f n +1 , va fi f n (iepurii nu mor niciodat !), la care se
adaug iepurii nou-nscui.Dar iepuraii se nasc doar din perechi de iepuri care au cel
puin o lun, deci vor fi f n 1 perechi de iepuri nou-nscui.
Obinem astfel o relaie de recuren:
f 0 = 0 , f1 = 1 , f n +1 = f n + f n 1 , care genereaz termenii irului lui Fibonacci.

Observaie. Acest ir exprim ntr-un mod naiv creterea populaiei de iepuri.Se


presupune c iepurii au cte doi pui o dat la fiecare lun dup ce mplinesc vrsta de
dou luni.De asemenea, puii nu mor niciodat i sunt unul de sex masculin i unul de
sex feminin.

46

Articole i note matematice

2. Fibonacci, numrul de aur, natura i arta.


2.1. Fibonacci i numrul de aur
Raportul de aur este un numr iraional(1.618033), putnd fi definit n diferite
moduri dar, cel mai important concept mathematic asociat cu regula de aur fiind irul lui
Fibonacci.Imprind orice

numr la predecesorul su, se obine aproximativ numrul de aur.Primii care l-au


folosit au fost egiptenii, majoritatea piramidelor fiind construite innd cont de numrul
de aur.Grecii au fost ns cei care l-au denumit astfel, folosindu-l att n arhitectur ct
i pictur i sculptur.Dealtfel numrul de aur se noteaz cu litera greceasc fi( ), de
la sculptorul grec Phidias.El a construit Parthenonul pornind de la acest raport.
S ncepem cu o problem estetic.S considerm un segment de dreapt.Care
este cea mai plcut mprire a unui segment n dou pri?Grecii antici au gsit un
rspuns pe care ei l considerau corect(teoreticienii l numesc simetrie dinamic).Dac
prii stngi a segmentului i atribuim lungimea u = 1 , atunci partea dreapt va avea o
lungime v = 0.618... Despre un segment partiionat astfel spunem c este mprit n
seciunea , sau proporia sau diviziunea de aur(divin).Ideea este c lungimea u
reprezint aceeai parte din tot segmentul (u + v) ct reprezint lungimea v din partea
u .Cu alte cuvinte:
u+v u
= .
u
v
u
Dac notm = , observm c:
v
1
u u+v u
1+ = 1+ =
= = , i este rdcina pozitiv a ecuaiei 2 1 = 0 ,
v
u
v

1+ 5
= 1.6180339887... Dac presupunem u = 1, atunci :
2
u 1
1+ 5
v = = = 1 =
= 0.6180339887...
2

Afirmm acum c este strns legat de irul lui Fibonacci.Aceasta este o ide
remarcabil a matematicii .
Mai observm c :
1
1
1
1
= 1+
= ... = 1 +
= 1+ = 1+
este o fracie infinit.
1
1
1

1+
1+
1+
1
1

1+
1+
1

1+
1 + ...
Dac privim fraciile pariale :

adic =

47

Articole i note matematice

1
1= ,
1

1 2
1+ = ,
1 1

1+

1+

1+

1
1+

1
1

3
,
2

1+
1+

1
1+

1
1

5
,
3

1+

1+
1+

8
,
5

1
1+

1
1

13
obsevm c toate rezultatele sunt rapoarte de numere
8

1+
1+

1
1
Fibonacci, fapt ce motiveaz teorema care spune c :
f
lim n +1 = .n cuvinte putem spune c, pe msur ce n se apropie de infinit, raportul
n f
n
termenilor al n + 1 lea i al n lea din irul lui Fibonacci se apropie de .
La fel de simplu cum este o fracie infinit, tot aa poate fi i un radical
infinit:
1+

= 1 + 1 + 1 + 1 + 1 + ... .Alt aplicaie a numrului apare la pentagonal


regulat deoarece :

2 cos( ) = i 2 sin( ) = 3 .
5
5
De asemenea exist o legtur ntre dreptunghiurile de aur i irul lui Fibonacci
deoarece lungimea i limea celui de-al n lea dreptughi pot fi scrise ca expresii
liniare, unde coeficienii sunt
ntotdeauna numere Fibonacci.Aceste dreptunghiuri pot fi nscrise ntr-o spiral
logaritmic .Spiralele logaritmice se ntlnesc destul de des n natur(carcasa unui melc,
colii unui elefant sau conurile de pin).Asemenea spirale sunt echiunghiulare, n sensul
1 + 3 3
,
) taie spirala sub un
c orice dreapt ce trece prin punctul ( x 0 , y 0 ) = (
5
5
unghi constant.
2.2. Fibonacci i plantele.
Plantele nu au cum s cunoasc numerele lui Fibonacci, dar se dezvolt n cel
mai eficient mod.
a. multe plante au aranjamentul frunzelor dispus ntr-o secven Fibonacci n jurul
tulpinei;

48

Articole i note matematice

b.anumite conuri de pin respect o dispunere dat de numerele lui Fibonacci ;


c.floarea soarelui are seminele dispuse dup o secven Fibonacci;

d.inelele de pe trunchiurile palmierilor respect numerele lui Fibonacci;


e.numrul petalelor florilor este, de cele mai multe ori, un numr al secvenei Fibonacci:
e.1.cala are 1 petal;
e.2.euphorbia are 2 petale;
e.3. irisul i crinul au 3 petale;
e.4.viorelele, lalelele, trandafirul slbatic i majoritatea florilor au 5 petale;
e.5.margaretele pot avea 21 de petale sau 34 de petale i exemplele sunt nenumrate;

e.6.florile cu un numr de petale care nu sunt n secvena Fibonacci sun rare i


considerate speciale.
Concluzia este realizarea unui optim, a unei eficiene maxime.Dac se urmeaz secvena
lui Fibonacci, frunzele unor plante pot fi dispuse astfel nct s ocupe un ct mai mic
spaiu i s obin ct mai mult soare.
49

Articole i note matematice

Ideea dispunerii frunzelor n acest sens pleac de la considerarea unghiului de aur de


222.5 grade; unghi care mprit la ntregul 360 de grade va da ca rezultat numrul
iraional 0.61803398, cunoscut ca raia irului lui Fibonacci.
2.3. Cochilia melcului, furnica i Fibonacci
Designul cochiliei melcului urmeaz o spiral foarte reuit, o spiral greu de
realizat cu pixul.Studiat n amnunt s-a ajuns la concluzia c aceast spiral urmrete
dimensiunile date de secvena lui Fibonacci:
pe axa pozitiv :1, 2, 5, 13, .a.m.d
pe axa negativ :0, 1, 3, 8, .a.m.d.

Se observ c aceste 2 subiruri combinate dau numerele lui Fibonacci.


i n acest caz raiunea i motivaia pentru aceast dispunere este simpl : n acest fel
cochilia i creaz melcului, n interior un maxim de spaiu i de siguran.

Furnica are corpul mprit n trei segmente, dup diviziunea de aur.

2.4. Fibonacci i corpul uman.


Faa uman este caracterizat, din punct de vedere estetic prin cteva dimensiuni
principale: distana ntre ochi, dintre gur i ochi i distana dintre nas i ochi,
dimensiunea gurii.n tiina esteticii se apreciaz c faa este cu att considerat mai
plcut ochiului cu ct aceste dimensiuni respect secvena lui Fibonaci mai bine.
De exemplu raportul dintre distana de la linia sursului(unde se unesc buzele) pn la
vrful nasului i de la vrful nasului pn la baza sa este aproximativ raportul de aur
Mna uman are 5 degete(numr din irul Fibonacci), fiecare deget avnd 3
falange separate prin 2 ncheieturi (numere din irul Fibonacci).Dimensiunile falangelor
sunt:2 cm, 3 cm, 5 cm.n continuarea lor este un os al palmei care are 8 cm.

50

Articole i note matematice

2.5. Fibonacci, numrul de aur i arta.


Dac privim lucrrile unor mari artiti, fie ei pictori, arhiteci, sculptori sau
fotografi, se observ c multe dintre ele au la baz regula de aur.Conform acesteia,
pentru ca un ntreg mprit n pri inegale s par frumos, trebuie s existe ntre
partea mic i cea mare acelai raport ca ntre partea mare i ntreg (Marcus Pollio
Vitruvius, arhitect roman).
Rudolf Arnheim(psiholog,s-a ocupat de psihologia artei) d o explicaie acestui
lucru astfel:Acest raport este considerat ca deosebit de satisfctor datorit modului n
care mbin unitatea cu varietatea dinamic.ntregul i prile sunt perfect proporionate,
astfel c ntregul predomin fr s fie ameninat de o scindare, iar prile i pstreaz
n acelai timp o anumit autonomie.(n Arta i percepia vizual).
n pictur a fost folosit mai ales n Renatere, probabil cea mai discutat utilizare
a acestuia fiind n tabloul lui Leonardo da Vinci, Mona Lisa.Capul, ca i restul
corpului e compus utiliznd raportul divin, cum i spunea da Vinci.n prima jumtate a
secolului trecut pictorul Piet Mondrian utilizeaz n picturile sale dreptunghiul de aur,
avnd raportul laturilor aproximativ 1.618De fapt, lucrrile sale sunt alctuite numai
din asemenea dreptunghiuri.Acest dreptughi este considerat cea mai armonioas form
geometric.Cu toate acestea, rareori este folosit pentru cadraje.Dac se mparte fiecare
latur a cadrului fotografic n 8 pri egale(numr din irul Fibonacci) i se unesc
punctele de pe laturile opuse corespunztoare diviziunilor 3 i 5 (numere din irul
Fibonacci) se obin aa numitele linii forte ale cadrului.Punctele aflate la intersecia
liniilor se numesc puncte forte.Practic se pot mpri laturile n trei pri egale, rezultatul
este aproximativ acelai.Se presupune c subiectul amplasat pe aceste linii sau n aceste
puncte determin o mprire armonioas a imaginii astfel nct ea nu este nici simetric,
nici plictisitoare, nici prea dezechilibrat.De exemplu, dou fotografii de Robert
Doisneau,Laccordioniste, 1951 i The cellist, 1957 i fotografia Poplar Trees a
lui Minor White n care toate liniile converg spre un punct forte.Ansel Adams se
mpotrivea regulilor, canoanelor.El spunea aa zisele reguli de fotocompoziie sunt
invalide , irelevante i imateriale; nu exist reguli de compoziie n fotografie, exist
doar fotografii bune.Cei mai muli fotografi ncalc regulile fotocompoziiei.Cu toate
acestea i n

imaginile lui se observ diviziunea de aur(vezi fotografia Aspens, 1958).Asta


nseamn c dei nu era de accord cu regulile le cunotea foarte bine.Dac fotografia are
valoare cu subiectul n centru, atunci nclcai regula diviziunii de aur!Subiectul trebuie
s fie n armonie cu celelalte elemente din cadru.Dac astfel se verific i diviziunea de
aur, este perfect!Toate acestea arat importana acestui numr, astfel c toi marii
fotografi au inut i in cont de el n conceperea unei fotografii.
51

Articole i note matematice

Pn i n muzic apare acest raport, se presupune c Bach sau Beethoven au


inut cont de el n compoziiile lor.
Atunci cnd scriei, ducei instinctiv linia din mijloc a literii E(la fel i cu A, F, B, R,)
2
de baz (aproximativ raportul de aur).
aproximativ la
3
Concluzie. Numerele lui Fibonacci sunt considerate a fi, de fapt, sistemul de numrare
al naturii, un mod de msurare al Divinitii, o legtur ntre matematic i art.

3. Unele rezultate referitoare la irul lui Fibonacci


Consultnd bibliografia enumerat am selectat urmtoarele rezultate:
Numerele lui Fibonacci f n sunt date de urmtoarea recuren :
f 0 = 0, f1 = 1 , f n +1 = f n 1 + f n , n 1 .
Teorema 1. Dac x 2 = x + 1 , atunci avem :
x n = f n x + f n 1 , n 2 .
Demonstraie.Vom demonstra prin inducie dup n .
Pentru n = 2 relaia este trivial .Presupunem c n > 2 avem x n 1 = f n 1 x + f n 2 .
Atunci
x n = x n 1 x = ( f n 1 x + + f n 2 ) x = f n 1 ( x + 1) + f n 2 x = ( f n 1 + f n 2 ) x + f n 1 = f n x + f n 1 .

Teorema 2.(Formula lui Binet).Termenul al n lea din irul lui Fibonacci este dat de:
1 1+ 5 n 1 5 n
fn =
((
) (
) ), n 0.
2
2
5

1+ 5
1 5
i 1 =
.
2
2
i (1 ) n = (1 ) f n + f n 1 .

Demonstraie.Rdcinile ecuaiei x 2 = x + 1 sunt =

Din Teorema 1., avem n = f n + f n 1

n continuare n (1 ) n = 5 f n , de unde rezult formula lui Binet.


Teorema 3. f1 + f 2 + ... + f n = f n + 2 1 .
Demonstraie. Avem relaiile:
f1 = f 3 f 2 , f 2 = f 4 f 3 , f 3 = f 5 f 4 ,..., f n = f n + 2 f n +1 , care prin adunare dau
f1 + f 2 + f 3 + ... f n = f n + 2 f 2 = f n + 2 1 .
Teorema 4. f1 + f 3 + f 5 + ... + f 2 n 1 = f 2 n .
Demonstraie. Observm c:
f1 = f 2 f 0 , f 3 = f 4 f 2 , f 5 = f 6 f 4 ,..., f 2 n 1 = f 2 n f 2 n 2 .Adunm
relaiile
obinem identitatea dorit.
2
2
2
2
Teorema 5. f1 + f 2 + f 3 + ... + f n = f n f n +1 .

Demonstraie. Avem f n 1 f n = ( f n +1 f n )( f n + f n 1 ) = f n +1 f n f n + f n +1 f n 1 f n f n 1 .
2

Atunci , obinem relaiile f n +1 f n f n f n 1 = f n , care prin adunare pentru n = 1,2,3,... ,


dau relaia final.
2
Teorema 6. (Identitatea lui Cassini). f n 1 f n +1 f n = (1) n , n 1.
Demonstraie.Observm c :

52

Articole i note matematice


2

f n 1 f n f n = ( f n f n 2 )( f n + f n 1 ) f n = f n 2 f n f n 1 ( f n 2 f n ) = ( f n 2 f n f n 1 )

Dac notm u n = f n 1 f n +1 f n , obinem u n = u n 1 i mai departe u n = (1) n 1 u1 .


2

Din cele de mai sus avem f n 1 f n +1 f n = (1) n 1 ( f 0 f 2 f 1 ) = (1) n .


n
n
Teorema 7.(Cesro). 2 k f k = f 3n .
k =0 k

Demonstraie.Utilizm formula lui Binet ,

n
n k k (1 ) k
n k
2

f
=
2
=

k

5
k =0 k
k =0 k
n

n
n
n
n
1
( 2 k k 2 k (1 ) k ) =
((1 + 2 ) n (1 + 2(1 )) n ).
k
k
5 k
5
k =0
2
3
Cum = + 1, obinem 1 + 2 = i similar 1 + 2(1 ) = (1 ) 3 .

Atunci , rezult

k 2

fk =

(( ) 3n + (1 ) 3n ) = 3n .

5

Teorema 8.(Vorobyov)Dac s 1, t 0 sunt ntregi atunci:
f s +t = f s 1 f t + f s f t +1 .
Demonstraie. Fixm pe t i demonstrm prin inducie dup s .Pentru s = 1 se obine
f t +1 = f 0 f t + f1 f t +1 , care este adevrat(trivial).Presupunem c s > 1 i c
f s k + t = f s k 1 f t + f s k f t +1 pentru orice k care satisfac 1 k s 1 .
Avem f s + t = f s +t 1 + f s +t 2
(din recurena Fibonacci)
= f s 1+t + f s 2+t
(trivial)
= f s 2 f t + f s 1 f t +1 + f s 3 f t + f s 2 f t +1 (din presupunerea fcut)
= f t ( f s 2 + f s 3 ) + f t +1 ( f s 1 + f s 2 )
(prin rearanjarea termenilor)
= f t f s 1 + f t +1 f s
(din recurena Fibonacci).
Observaia 1. J.R. Silvester indic o metod elegant care furnizeaz identiti pentru
0 1
i se constat c
termenii irului ( f n ) . Mai precis, se consider matricea A =
1
1

k =0

fn
f
, n = 1,2,... .Plecnd de la aceast observaie i utiliznd
A n = n 1
f n +1
fn
n
egalitile A n + m = A n A m , n, m = 1,2,... i det( A n ) = [det( A)] , rezult relaia din
eorema 6. i de asemenea relaia din teorema 8.
Posibilitile de a obine identiti, folosind ideea de mai sus sunt multiple.Astfel,
observm c A n = f n A + f n 1 I (cu I am notat matricea untate), iar pentru n = 2 ,

avem

obinem: A 2 = A + I .De asemenea avem A n + 2 = A n +1 + A n .Enunate fiind aceste


proprieti ale matricei A , se observ c puterile acesteia verific recurene de tip
Fibonacci.Deoarece A I = I A = A , putem aplica formula binomului lui Newton
pentru A i I , apoi identificnd relaiile obinute pe componente se obin diferite
identiti.
Prezentm mai jos, fr demonstraie, cteva identiti obinute prin metoda expus mai
sus:

53

Articole i note matematice


n
n
n
n
n
n
(1) f 2 n = f k ; (2) f 2 n +l = f k +l ; (3) f l = (1) k f 2 n k +l ;
k =0 k
k =0 k
k =0 k
n
n
n
n
(4) f l + n = (1) k f 2 n 2 k + l ; (5) f 3n + l = 2 k f k +l (teorema
lui
k =0 k
k =0 k
generalizat);

Cesro

n
n
n
n
(6)2 n f n +l = (1) k f 3n 3k +l ; (7) f l = 2 k (1) k f 3n 2 k +l ;
k =0 k
k =0
k
m
m mk
k
(8) f nm = f n f n 1 f m k .
k =0 k
Pentru studeni propun demonstrarea urmtoarelor identiti:

fn
1
1
(9)
= 1 ; (10)
= 1 ; (11)
= 4 ;
n = 2 f n 1 f n +1
n =1 f n +1 f n + 2
n =0 f 2n

(12) arctan
n =1

f 2 n +1

; (13) lim
n

fn

1
5

; (14) lim
n

f n+ r
=r.
fn

Observaia 2. Unii autori, au obinut identiti cu termenii irului lui Fibonacci cu


ajutorul determinanilor.Astfel, dac considerm irul lui Fibonacci : 1, 2, 3, 5,se
2 1 0 0 ... ... 0
1 2 1 0 ... ... 0
observ c f n = Dn 1 = 1 1 2 1 0 ... 0 este deci dat de un determinant de
... ... ... ... ... ... ...
1 1 1 ... ... 1 2
ordinul n 1 .
2 1 0 0 ... ... 0
1 2 1 0 ... ... 0
Dac considerm determinantul de ordinul n , Dn = 1 1 2 1 0 ... 0
... ... ... ... ... ... ...
1 1 1 ... ... 1 2
pe care l dezvoltm dup elementele primei linii ne d
:
1 1 0 0 ... ... 0
1 2 1 0 ... ... 0
Dn = 2 Dn 1 1 1 2 1 0 ... 0 .Dac dezvoltm i ultimul termen obinem

... ... ...


1 1 1
1
1
Dn = 2 Dn 1 Dn 2 + 1
...
1

...
...
1
2
1
...
1

...
...
0
1
2
...
1

...
1
0
0
1
...
...

...
2
...
...
0
...
...

...
...
...
...
1

0
0
0 .Procedm ca mai sus i deducem :
...
2
54

Articole i note matematice

1 1 0 0 ... ... 0
1 2 1 0 ... ... 0
Dn 1 = 2 Dn 2 1 1 2 1 0 ... 0 .Dac adunm membru cu membru ultimile
... ... ... ... ... ... ...
1 1 1 ... ... 1 2
dou egaliti obinem : Dn + Dn 1 = 2 Dn 1 Dn 2 + 2 Dn 2 Dn = Dn 1 + Dn 2 ,
relaie de recuren analoag cu cea din irul lui Fibonacci f n +1 = f n 1 + f n .

Teorema 9. f n f nk , n, k N .
Demonstraie. Prin inducie dup k N i orice n N .
Pentru k = 1 f n = f n1 f n f n (adevrat).

Presupunem f n f nk i demonstrm c f n f n ( k +1) .


ntradevr, innd seama de teorema 8. avem: f n ( k +1) = f nk + n = f nk +1 f n + f nk f n 1 ,
n N i deoarece f n f n i f

f nk rezult f n f n ( k +1) .

Teorema 10. f kn 1 f n 1 (mod f n ) , k , n N .


k

Demonstraie.Se arat tot prin inducie dup k N .Pentru k = 1 relaia este


evident.Pentru k = 2 inem seama de teoremele precedente i avem ;
2

f 2 n 1 = f n f n + f n 1 f n 1 f n 1 (mod f n ), n N .Fie

f kn 1 f n 1 (mod f n ) .Avem
k

deci c : f ( k +1) n 1 = f kn1+ n = f kn f n + f kn 1 f n 1 f kn 1 f n 1 f n 1 f n 1 f n 1

k +1

(mod f n ) i

deci conform principiului induciei complete relaia este demonstrat.


2
k
Teorema 11. f kn 1 (1) k +1 f n 2 (mod f n ), n N .
Demonstraie.Relaia se demonstreaz tot prin inducie complet .Pentru k = 1 obinem
2
f n 2 f n 2 (mod f n )
ceea
ce
este
evident.Presupunem
c
k

f kn 2 (1) k +1 f n 2 (mod f n )

i s demonstrm c f ( k +1) n 2 (1) k + 2 f n 2

k +1

(mod f n ) .ntradevr, avem:


k

f ( k +1) n 2 = f kn 2+ n = f kn 1 f n + f kn 2 f n 1 f kn 1 f n + (1) k +1 f n 2 ( f n f n 2 )
k

f kn1 f n + (1) k +1 f n 2 f n + (1) k + 2 f n 2


(1) k + 2 f n 2

k +1

k +1

( f n 1 + (1) k +1 f n 2 ) f n + (1) k + 2 f n 2

(mod f n ).
k

Am folosit mai sus c f n 1 + (1) k +1 f n 2 se divide cu f n 1 + f n 2 = f n .


Rezult conform principiului induciei complete c teorema este demonstrat.
2
Teorema 12. f n f nf n , n N .
Demonstraie.Notm
k
k
f n = k .Avem: f nf n = f nk = f nk 1 + f nk 2 f n 1 + (1) k +1 f n 2 (mod k 2 ).

Totodat avem:

55

k +1

Articole i note matematice

f n 1 = f n f n 2

k
k
k
k i
i
k
f n 1 = ( f n f n 2 ) k = (1) i f n f n 2 (1) k f n 2 (mod k 2 ).
i
i =0
k
k
k
k
k +1
f n 1 + (1) f n 2 (1) k f n 2 + (1) k +1 f n 2 0(mod k 2 ) ,
Deci,
demonstreaz teorema.

Teorema 13. f n

m +1

deci

ceea

ce

f nf m , m N i n N .
n

Demonstraie.Pentru m = 1 afirmaia este echivalent cu teorema 12.Presupunem


afirmaia adevrat pentru pentru m i o demonstrm pentru m + 1 .Vom arta c
fn

m +1

f nf m f n
n

m+ 2

f nf m +1 .Notm i atunci avem:


n

f uf n = f uf n 1 + f uf n 2 f u 1 n + (1) f n +1 f u 2 n (mod u 2 ) ( f u f u 2 ) f n + (1) f n +1 f u 2 n (mod u 2 )


f

(1) f n f u 2
m

fn u2 fn

fn

+ (1) f n +1 f u 2

m +1

fn

0(mod f n

m +1

) .Am folosit mai sus c din f n u i

fu .

Teorema 14.Orice dou numere Fibonacci consecutive sunt relative prime.


Demonstraie.Fie d = ( f n , f n +1 ). Avem f n +1 f n = f n 1 de unde rezult d f n 1 .Atunci
d ( f n f n 1 ) = f n 2 .Repetnd procedeul se deduce c d f1 , deci d = 1 .
2

Altfel:din teorema 6., f n 1 f n +1 f n = (1) n . Rezult d (1) n , i.e., d = 1 .


Teorema 15. ( f m , f n ) = f ( m ,n ) .
Demonstraie.Notm a = (m, n), b = ( f m , f n ), c = f ( m,n ) .Vom arta c c b i b c .

Deoarece a m i a n , conform teoremei 9. avem: f a f m i f a f n .Deci f a ( f m , f n ) , i.e.,


cb.

Acum, din Teorema Bachet-Bezout, exist numerele ntregi x, y astfel nct


xm + yn = a .Se observ c x i y nu pot fi ambele negative, deoarece a ar fi
negativ.Cum a n i a m , avem a n, a m .De asemenea, x i y nu pot fi simultan
pozitive , deoarece am avea a = xm + yn m + n , contradicie.Atunci , x i y au
semne diferite i, fr a restrnge generalitatea presupunem c x 0 , y > 0 .
Observm c :

f yn = f a xm = f a 1 f xm + f a f xm +1 (am utilizat teorema 8.).


Cum n yn i m ( xm) , din teorema 9. rezult c f n f yn i f m f xm .Acestea implic c
( f m , f n ) f yn i ( f m , f n ) f xm .Din cele de mai sus avem c ( f m , f n ) f a f xm +1 .
Dac ( f m , f n ) f xm +1 , cum ( f m , f n ) f xm rezult c ( f m , f n ) ar divide dou numere

Fibonacci consecutive, contradicie( conform teoremei 14 ) n cazul n care


( f m , f n ) > 1.
Cazul ( f m , f n ) = 1 este trivial.Rezult c ( f m , f n ) f a , ceea ce trebuia demonstrat.
56

Articole i note matematice


Teorema 16.Dac p 5 este un numr prim impar, atunci p f p 1 sau p f p +1 .
Demonstraie.
p 1
(1) n mod p,1 n p 1.
Lema 1.
n
Demonstraie. ( p 1)( p 2)...( p n) (1)(2)...(n) (1) n n!mod p ,
de
concluzia.
p + 1
0 mod p,2 n p 1.
Lema 2.
n

Demonstraie. ( p + 1)( p)( p 1)...( p n 2) (1)(0)(1)...(n) 0 mod p ,ceea


demonstreaz lema.
Din teorema 2. avem:
n2
n
1 n n 2 n
) .
f n = n 1 ( + 5 + 5 + ... + 5 2
2
1 3
5
n 1
Din lema 1.,
2 p 2 f p 1 p 1 (5 + 5 2 + ... + 5

p 3
2

p 1
2

aici

ce

mod p .

Din lema 2.,


2 p f p +1 p + 1 + 5

p 1
2

(5

p 1
2

+ 1) mod p.

Din cele dou relaii de mai sus obinem:


2 2 p f p 1 f p +1 (5 p 1 1) mod p .
Din mica teorem a lui Fermat, 5 p 1 1mod p , pentru p 5 i teorema este
demonstrat.

Not. Punctul de plecare al acestui articol l-a constituit rspunsul dat de dl. prof. dr.
Ioan Tomescu (Membru Corespondent al Academiei), Secretarului General al S.S.M.R
din Romnia dl. prof. Mircea Trifu, n Gazeta Matematic nr.12 / 2007, la ntrebarea :
M.T.:Mai sunt dispui tinerii de astzi s nvee matematica?
I.T.:Dac vom ti s prezentm aceast tiin ca pe o frumoas provocare a spiritului
mereu nscocitor, este posibil ca tinerii s ajung s nteleag frumuseea i
profunzimea unui raionament matematic.i mai trebuie ca profesorii s fie capabili s
prezinte elevilor impactul matematicii asupra ntregii dezvoltri tiinifice
contemporane, conexiunile dintre matematic i informatic i aplicaiile acestora, de
exemplu, n criptografie, n studiul genomului uman, n comerul electronic.

Bibliografie

[1] Gazeta Matematic, 1895 2007 .


57

Articole i note matematice

[2] Fauvel, J., & van Maanen, J., History in Mathematics Education, Boston, 2000.
[3] Finch,S.R., Mathematical Constants, Cambridge University, 2003.
[4] Knot, R., Fibonacci Numbers and the Golden Section, se poate consulta gratuit pe
Internet la adresa http://www.mcs.surrey.ac.uk/Personal/R.Knot/Fibonacci/fib.html
[5] Mihileanu, N., Istoria Matematicii, vol. 1, Editura Enciclopedic Romn,
Bucureti, 1974.
[6] Mihileanu, N., Istoria Matematicii, vol. 2, Editura tiinific i Enciclopedic,
Bucureti, 1981.
[7] Vorobyov, N.N., The Fibonacci Numbers, The University of Chicago, 1966.
[8] Santos, D.A., Number Theory for Mathematical Contests, Boston, 2007.
[9] Silvester, J.R., Fibonacci properties by matrix methods, The Mathematical Gazette,
vol. 63 (1979), nr. 425, pp. 188 191.
[10] Weisstein, E.W., CRC Concise Encyclopedia of Mathematics, Washington, D.C.,
2003.
Grupul colar Tehnic Sf. Mucenic Sava, Berca, Buzu

DIVIZIUNI I FASCICULE ANARMONICE


DE NECULAI STANCIU
Geometria proiectiv este toat geometria
Arthur Cayley

I.

DIVIZIUNI ANARMONICE

58

Articole i note matematice

Aceast articol, este dedicat geometriei sintetice (fundamentelor geometriei i


geometriei proiective), i se adreseaz studenilor, elevilor, profesorilor de matematic,
i n general, oricui este interesat de geometrie.
n prezent, geometria din liceu este introdus via algebra liniar (adic vectorial
nu sintetic).Articolul, vine s detalieze noiunea de diviziune i fascicul armonic definit
n articolul -Aplicaii ale teoremei lui La Hire - G.M. - 3 / 2008 ; noiuni care pot fi
nelease de orice elev din clasa a VI-a.
Diviziunea i fasciculul armonic, sunt tratate n capitole de fundamentele
geometriei, i n geometria proiectiv, unde metoda utilizat este cea axiomatic
sintetic.Alte noiuni legate de acestea sunt:polaritate, dualitate, etc.
Considerm punctele coliniare A, B i C ca n fig.1 .
x

Fig.1
Avem relaiile: (1) AB = BA i (2) AB + BC = AC ; AB + BC + CA = 0 .Fie acum patru
CA
DA
.
puncte coliniare A, B, C i D ca n fig. 2, astfel nct (3)
=
CB
DB
x

Spunem c perechea (CD) este armonic


conjugat cu perechea ( AB) i reciproc
Fig.2 deoarece:
x

AC BC CA CB CA DA
:
=
:
=
:
= 1 .Se mai spune c A este conjugatul armonic al
AD BD DA DB CB DB
lui B n raport cu C i D sau, B este conjugatul armonic al lui A n raport cu C i
D sau, C este conjugatul armonic al lui D n raport cu A i B sau, D este conjugatul
armonic al lui C n raport cu A i B .
n continuare vom numi biraport sau, raport anarmonic sau, diviziune anarmonic
not
CA DA not
raportul (4)
:
= ( ABCD) = r .Se observ c daca r = 1 atunci ( ABCD) este
CB DB
diviziune armonic.

Teorema I.1. Raportarea la o origine de pe ax.


x
O
A
B
x
Dac avem axa x x cu originea O i punctele
x

A
A

O
B

B
O

A i B pe ax atunci (5) AB = OB OA .

Demonstraie.Avem cazurile din fig. 3.

Fig.3

Cazul O A B . OA + AB + BO = 0
59

Articole i note matematice


AB = BO OA = OB OA .
Cazul A O B .Din relaia (2) avem AO + OB + BA = 0 AB= OB + AO = OB OA .
Cazul A B O . AB + BO + OA = 0 AB = BO OA = OB OA .
Cuarte de punte coliniare.
x
A B C D
x Avem configuraia din fig. 4:
AB i CD sunt segmente care se separ;
AD i BC sunt segmente care se includ iar;
Fig.4
AC i BD sunt segmente care se ncalec.

Teorema I.2. Echipolena lui Euler.


(6) AB CD + AD BC = AC BD
Demonstraie.
AB CD + AD BC AC BD = AB( AD AC ) + AD( AC AB) AC ( AD AB) =
= AB( AD AC AD + AC ) + AD( AC AC ) =0.
Teorema I.3.Punctul care mparte un segment ntr-un raport dat.
AM
Fie segmentul AB , M AB , O AB origine aleas arbitrar i k =
.
AB
Avem (7) OM = (1 k )OA + kOB .
Demonstraie.Presupunem ordonarea O A M B ,cu AM = k AB .Rezult imediat din
teorema 1, OM OA = k OB k OA .Celelalte ordonri se trateaz analog.
1
OA + OB
Consecin.Dac M este mijlocul segmentului AB , atunci k = i OM =
.
2
2
Teorema I.4.Unicitatea punctului care mparte un segment ntr-un raport dat.
Fie pe axa x x , segmentul AB i punctele C , D n interiorul sau n exteriorul segmentului
CA DA
AB dar de aceeai parte a lui.Dac
atunci D = C .
=
CB DB
Demonstraie.Considerm ordonarea A C D B .
AC DB = AD CB .Din
Din
ipotez
rezult
(6)
avem
AD CB = AC DB + AB CD .Rezult AB CD = 0 , de unde, deoarece AB 0 obinem
CD = 0 , adic C = D .
n cazul n care C i D sunt ambele exterioare i situate de aceeai parte a segmentului
AB , se procedeaz analog.Cazul n care C i D sunt separate de segmentul AB , adic
ordonarea
C A B D sau D A B C se exclude deoarece AB 0 .
Teorema I.5.Diviziuni anarmonice egale, cu trei perechi de puncte comune.
Dac ( ABCD) = ( ABCD ) , atunci D = D .
def
DA D A T 4
CA DA CA D A
Demonstraie. ( ABCD) = ( ABCD )
:
=
:

=
D = D .
( 4 ) CB DB
DB D B
CB D B
(q.e.d).
Teorema I.6. Valoarea biraportului r nu poate fi nici 0 nici 1.
CA DA
Demonstraie. r = ( ABCD) =
:
0
deoarece punctele fiind distincte
CB DB
CA 0, CB 0, DA 0, DB 0 .

r 1 =

CA DA
CA DB
AC BD
AC BD BC AD T 2 AB CD
:
1 =

1 =
1 =
=
0
CB DB
CB DA
BC AD
BC AD
BC AD
60

Articole i note matematice


r 1.
n continuare, vom calcula valorile biraportului obinute prin permutarea punctelor
unei diviviuni anarmonice.
Teorema I.7.O transpoziie a dou puncte din aceeai pereche inverseaz raportul
anarmonic.
CA DA CA DB

r = ( ABCD) = CB : DB = CB DA
1
Demonstraie.
r r1 = 1 r1 = .
r
r = ( ABDC ) = DA : CA = DA CB
1
DB CB DB CA
Am
demonstrat
1
CB DB CB DA
1
:
( ABDC ) =
. ( BACD) =
=
.(q.e.d).
=
CA DA CA Db ( ABCD)
( ABCD)
Teorema I.8.O transpoziie a dou puncte din perechi diferite ne d un raport
anarmonic complementar .
BA DA
CA DA AC BD
Demonstraie. r = ( ABCD) =
:
=
i r2 = ( ACBD) =
:
=
BC DC
CB DB BC AD
AB CD
.Summ cele dou egaliti i aplicnd echipolena lui Euler(teorema 2)
=
BC AD
AC BD AB CD AD BC
obinem: r + r2 =
=
= 1 r2 = 1 r ( ACBD) = 1 r .
BC Ad
AD BC
Analog se arat c ( DBCA) = 1 r .
Transformrile precedente ale raportului anarmonic, definite n teoremele I.7 i I.8,
compuse succesiv cu ele nsele l reproduc pe r .
Adic :
1
1
r1 = r = = r , respectiv r2 = 1 r r = 1 r2 = r .
r
r1
Vom numi transpoziii complementare acele transformri care compuse cu ele nsele ,
las raportul anarmonic neschimbat.
Teorema I.9. Dac ( ABCD) = ( ABDC ) sau ( ABCD) = ( BACD) atunci ( ABCD) = 1 .
TI .7
1
1
Demonstraie.Fie r = ( ABCD) ( ABDC ) = , ( BACD) = .
r
r
r 1
1
Dac r = r 2 1 = 0 r = 1 .(q.e.d).
r
Deoarece cu 4 obiecte putem face 24 de permutri, pentru fiecare diviziune de 4 puncte
avem 24 de valori ale rapoartelor anarmonice corespunztoare cte unei permutri.
Dintre toate acestea numai 6 valori sunt distincte, iar restul se obin prin transpoziii
complementare .Dintr-o permutare dat putem selecta alte trei de acelai raport
anarmonic.Obinem:
( ABCD) = ( BADC ) = (CDAB) = ( DCBA) = r
1
( ABDC ) = ( BACD) = ( DCAB) = (CDBA) =
r
( ACBD) = (CADB) = ( BDAC ) = ( DBCA) = 1 r
r 1
( ADBC ) = ( DACB) = (CBDA) = ( BCAD) =
r
1
( ACDB) = (CABD) = ( BDCA) = ( DBAC ) =
1 r

61

Articole i note matematice

( ADCB) = ( DABC ) = ( BCDA) = (CBAD) =

r
.
r 1

II. FASCICULE ANARMONICE

Considerm fig.1 unde S (a, b, c, d ) sau S ( A, B, C , D) reprezint un fascicul convergent,


cu punctul S propriu de raze a, b, c, d sau SA , SB, SC , SD i fig.2 n care S (a, b, c, d )
este un fascicul paralel de raze a, b, c, d sau SA , SB, SC , SD cu punctul S impropriu.

Fig.1
Fig.2

Dac diviziunea ( ABCD) este diviziune armonic atunci fasciculul ataat S ( ABCD) se
numete fascicul armonic.
Biraportul ataat unui fascicul convergent.
Fie fasciculul S (abcd ) tiat de secanta (vezi fig.1) n punctele
not

A = a, B = b, C = c, D = d .Dac S ( XYZ ) = aria triunghiului de vrfuri


not

CA CA h 2 S (CSA) S (CSA)
X , Y i Z , XY = XSY , h = d ( S , ) , atunci
=
=
.
=
CB CB h 2 S (CSB ) S (CSB )

CA DA S (CSA) S ( DSA) SC SA sin(ca) SD SA sin( da)


=
=
=
( ABCD) =
:
:
:

CB DB S (CSB) S ( DSB) SC SB sin(cb


) SD SB sin(db)

sin(ca) sin(da)
=
.
:

sin(cb) sin(db)

62

Articole i note matematice

sin(ca) sin(da )
Dac S (abcd ) =
, atunci rezult ( ABCD) = S (abcd ) .
:

sin(cb) sin(db)
Teoreme de invarian
Teorema II.1.Fiind date pe dreapta , punctele fixe A, B, C , D .Pentru orice S ,
notm a = SA, b = SB, c = SC , d = SD .Biraportul ataat fasciculului S (abcd ) este
invariant.
Demonstraie.Fie S , S , ca n fig.3.
not

Fig.3

S
S

A
a

S (abcd ) = ( ABCD)
S (a bc d ) = ( ABCD)
Din
i
rezult
S (abcd ) = S (a bc d ) .(q.e.d).
Teorema II.2.Fiind dat fasciculul fix de vrf S i raze a, b, c, d .Pentru orice secant
care intersecteaz razele fasciculului n A = a , B = b , C = c , i D = d ,
biraportul ataat diviziunii ( ABCD) este invariant.
Demonstraie.Fie i dou secante oarecare (fig.4), care intersecteaz razele
fasciculului n punctele A, B, C , D i A, B , C , D .

Fig.4

S
A

C B D

63

Articole i note matematice

Avem
( ABCD) = S (abcd )
i
( ABCD) = ( AB C D ) .(q.e.d.).
Fasciculul tiat de o secant paralel cu una din raze.
Fie fasciculul S (abcd ) i a (fig.5).

( AB C D ) = S (abcd ) .Rezult

Fig.5

D
C
64

Articole i note matematice

B
a

C A D A
C A SA
:
,(2) C AS C BC
,
=
C B D B
C B CB
D A SA
(3) D AS D BD
=
.Din (1),(2) i (3) rezult :
D B DB
SA SA
1
1
S (abcd ) =
:
=
:
= ( ABC D ) .
CB DB CB DB

(1) S (abcd ) = ( ABC D ) =

1
1
:
.
CB DB
Deoarece a scriem a = Ai (punctul impropriu pe direcia paralelelor a ),
Avem urmtoarea regul mnemotehnic pentru scrierea valorii biraportului

S (abcd ) =

CAi DAi
i luom CAi : DAi = 1 (trecerea la limit A Ai ).
:
CB DB

1
1
:
.
CB DB
Consecin.Fie B, C , D puncte fixe pe dreapta , a , S a , SB = b, SC = c, SD = d .

S (abcd ) =

Atunci S a, S (abcd ) este invariant.


CAi DAi
1
1
:
=
:
=constant.
CB DB CB DB
Teorema II.3.Fie A, B, C , D puncte fixe pe C (O; R) i M C (O; R) (fig.6).Dac
MA = a, MB = b, MC = c, MD = d atunci , M C (O; R) M (abcd ) este invariant.

Demonstraie.Fie Ai = a . S (abcd ) = ( Ai BCD ) =

sin(ca) sin(da )
Demonstraie. M (abcd ) =
=constant, deoarece A, B, C , D sunt puncte
:

sin(cb) sin(db)

CB
DCBA
DCB
CBA
fixe i ca =
=ct., cb =
=ct., da =
=ct., db =
=ct.
2R
2R
2R
2R

65

Articole i note matematice

Observaie. Din figura 6 rezult M ( ABCD) = M ( AB C D ) .


Teorema II.4. Fie A, B, C , i D puncte fixe pe C (O; R) iar , a, b, c, i d tangentele n
cele patru puncte la cercul C (O; R) .Atunci oricarea ar fi tangenta t la cercul C (O; R) n
punctul T C (O; R) , punctele A1 = a t , B1 = b t , C1 = c t i D1 = d t formeaz
o diviziune anarmonic ( A1 B1C1 D1 ) invariant.
Demonstraie.Considerm fig.7.

66

Articole i note matematice

Fig.7
Avem ( A1 B1C1 D1 ) = O( A1 B1C1 D1 ) .Formm apoi fasciculul cu vrful n T i raze
TA OA1 , TB OB1 , TC OC1 i TD OD1 .Deci T ( ABCD ) = O( A1 B1C1 D1 ) .

CBA
CB
DA
BCD
: sin
) : (sin
: sin
) =constant.
Obinem ( A1 B1C1 D1 ) = T ( ABCD ) = (sin
2R
2R
2R
2R
Teorema II.5.Pe cercul C (O; R) considerm punctele distincte A, B, C , D i tangentele
a, b, c, d n aceste puncte la cerc (fig.8).Avem egalitile:
A(aBCD) = B( AbCD) = C ( ABcD) = D( ABCd ) .

Demonstraie. A(aBCD) = (sin CAa : sin CAB) : (sin DAa : sin DAB) =

CDA
BC
DA
BCD
: sin
) : (sin
: sin
) = r = constant=
= (sin
2R
2R
2R
2R
= B( AbCD) = C ( ABcD) = D( ABCd ) (din egaliti de sinusuri).

67

Articole i note matematice

Fig. 8
Observaie. Teorema II.5 reprezint cazul limit a teoremei II.3 n care punctul M de
pe C (O; R) coincide cu unl din punctele A, B, C sau D .
Teorema II.6.Pe cercul C (O; R) se consider punctele distincte A, B, C , D i
tangentele la cerc n aceste puncte: a, b, c, d (fig.9).
Dac notm E = a b, F = b c, G = c d , H = d a, I = b d i J = a c, atunci
avem egalitile: ( AEJH ) = ( EBFI ) = ( JFCG) = ( HIGD) .

68

Articole i note matematice


Demonstraie.Considerm fasciculul cu vrful n O i raze OA, OE , OJ i OH

J
Fig.9
OAEJH , apoi fasciculul cu vrful n A i razele perpendiculare pe razele fasciculului
anterior : a OA, AB OE , AC OJ respectiv AD OH , A(aBCD) i avem :
( AEJH ) = O( AEJH ) = A(aBCD) .
Analog se obin i relaiile:
( EBFI ) = O( EBFI ) = B( AbCD) ;
( JFCG) = O( JFCG) = C ( ABcD) ;
( HIGD) = O( HIGD) = D( ABCd ) .
Acum aplicm relaii ntre sinusuri, ca n teorema 5, pe care aici le detaliem astfel:

CDA
ABC
CDA
CAa = CBA = cCA =
=
i CDA =
, rezult:
2R
2R
2R

CDA
sin(CAa ) = sin(CBA) = sin(cCA) = sin(CDA) = sin(
) i analoagele
2R

CB
sin(CAB ) = sin(CBb) = sin(cCB) = sin(CDB ) = sin( )
2R

DA
sin( DAa) = sin( DBA) = sin( DCA) = sin( dDA) = sin(
)
2R

DAB
sin( DAB) = sin( DBb) = sin( DCB ) = sin( dDB) = sin(
).
2R
innd seama de aceste valori putem scrie:

69

Articole i note matematice

CDA
CB
DA
DAB
A(aBCD ) = B( AbCD ) = C ( ABcD) = D( ABCd ) = (sin
: sin
) : (sin
: sin
)
2R
2R
2R
2R

(q.e.d.).
Observaie. Teorema II.6 reprezint cazul limit al teoremeiII. 4 n care tangenta t la
cercul C (O; R) coincide cu una din tangentele a, b, c, d .
Teoreme de concuren i coliniaritate.
Teorema II.7. Dac dou diviziuni anarmonice egale ( ABCD) = ( AB C D ) au un
punct comun A , atunci dreptele BB , CC i DD sunt concurente.
Demonstraie.Avem fig.10.
O

Fig.10

B
A

C
C

Fie O = BB CC i D = OD .Din teorema II.2 rezult ( ABCD) = ( AB C D )


iar, din ipotez avem ( ABCD) = ( AB C D ) .Deci ( AB C D ) = ( AB C D ) , apoi din
teorema I.5 se obine D = D .(q.e.d.).
Teorema II.8.Dac dou fascicule anarmonice egale S (abcd ) = S (ab c d ) au o raz
comun SS = a , atunci punctele de intersecie ale celor trei perechi de raze
corespondente: B = b b, C = c c , D = d d sunt coliniare.
Demonstraie.

70

Articole i note matematice

Fig.11

c
B

Fie A = BC a, D = BC d i D = DC d (fig.11).
Din ipotez rezult (1) S (abcd ) = S (abc d ) .Intersectnd fasciculul S (abcd ) cu
secanta BC rezult (2) S (abcd ) = ( ABCD) .Inersectnd fasciculul S (ab c d ) cu
secanta BC rezult (3) S (abc d ) = ( ABCD ) .Din cele trei relaii avem
( ABCD) = ( ABCD ) , apoi din teorema I.5. se obine D = D .(q.e.d.).
Teoreme clasice.
Teorema II.9. Prima teorem a lui Papus.
n ABC fie A, B , C trei puncte coliniare situate pe laturile BC , CA, AB i
BB CC = M , AM BC = A1 .n aceste condiii A i A1 sunt conjugate armonic n
raport cu B i C , adic ( BCAA1 ) = 1 (fig.12).
Demonstraie.
Fig.12

A
C
B

N
M

B
A1

Fsciculul de vrf M i raze MB, MC , MA, MA1 l tiem mai nti cu secanta BC i apoi
cu secanta B C i aplicm teorema II.2.
(1) M ( BCAA1 ) = ( BCAA1 ) = ( B C AN ) .

71

Articole i note matematice

Fasciculul de vrf A determinat de diviziunea anarmonic ( B C AN ) l tiem cu


secanta BC i aplicm teorema II.2.
(2) ( B C AN ) = A(CBAA1 ) = (CBAA1 ) .
Din (1) i (2) avem ( BCAA1 ) = (CBAA1 ) apoi din teorema I.9. rezult
( BCAA1 ) = 1 .(q.e.d.).
Teorema II.10. A doua teorem a lui Papus.
Pe dou drepte i luom trei iruri de puncte arbitrare A, B, C respectiv
A, B , C .Interseciile U = BC B C ,V = CA C A i W = AB AB sunt trei
puncte coliniare (fig.13).
Demonstraie.
a) cazul dreptelor incidente( )

C
B
A
W
O

Y
V
B

U
C

Fig.13

Fie X = B A AC i Y = B C = C A .
Considerm fasciculele de vrfuri A i C ale cror raze trec prin punctele diviziunii
(OABC ) .Din teorema II.1 rezult (1) A(OABC ) = C (OABC ) .Intersectnd primul
fascicul cu secanta B A , din teorema II.2. rezult (2) A(OABC ) = ( B AWX ) .Analog,
intersectnd al doilea fascicul cu secanta B C ,din teorema II.2. rezult (3)
C (OABC ) = ( B YUC ) .
Din (1),(2) i (3) rezult ( B AWX ) = ( B YUC ) , diviziuni anarmonice egale, cu B punct
comun.Conform teoremei II.7 i schemei de mai jos:
Puncte corespondente
B
B punct comun
A
Y dreapta AY
W
U dreapta WU
X
C dreapta XC
Rezult AY XC = AC CA = V V UW .(q.e.d).
b) cazul dreptelor paralele ( )

72

Articole i note matematice

B
X
W
A

Y
V

U
B

Fig.14

Considerm fasciculele de vrfuri A i C :


(1) A( ABC ) = C ( ABC ) ; (2) A( ABC ) = ( B AWX ) ;(3) C ( ABC ) = ( B YUC ) .
Rezult ( B AWX ) = ( B YUC ) , cu B punct comun, analog ca n cazul a) deducem
V UW .(q.e.d.).
Teorema II.11. Teorema plan a lui Desargues, direct i reciproc.
Dac dou triunghiuri au vrfurile aezate pe trei drepte concurente atunci laturile
lor corespunztoare se taie n trei puncte coliniare i reciproc dac intersecia
perechilor de laturi a dou triunghiuri sunt coliniare atunci vrfurile corespunztoare
sunt aezate pe trei drepte concurente.
Avnd n vedere c teorema rmne valabil i n cazul n care punctul de
concuren al celor trei drepte este impropriu iar unul sau toate cele trei puncte de
intersecie pot fi improprii enunul poate fi reformulate astfel:
Fie trei drepte a, b, c pe care luom : A, A a ; B, B b ; C , C c .Dreptele a, b, c
sunt concurente n O (propriu sau impropriu) dac i numai dac interseciile
A0 = BC B C , B0 = CA C A i C 0 = AB AB sunt situate pe o dreapt
(proprie sau improprie).Spunem c ABC i AB C sunt omologice , O este
centrul omologiei i = B0 C 0 este axa omologiei.
Demonstraie.
Necesitatea.Fie D = b AC , D = b AC .Considerm fasciculul O(abcOB0 ) tiat de
AC respectiv de AC (fig.15). Din teorema II.2., avem relaiile:
(1) ( AD C B0 ) = ( ADCB0 ) ;
(2)
( ADCB0 ) = B( AbCB0 ) ;
(3)
( AD C B0 ) = B ( AbC B0 ) .
Din relaiile de mai sus rezult B ( AbCB0 ) = B ( AbC B0 ) , cu raz comun.
Din teorema II.8. i schema de mai jos:
Raze corespondente
BA
B A
BA B A = C 0
b
b
raz comun
BC
B C
BC B C = A0
BB0
B B0
BB0 B B0 = B0
rezult A0 , B0 , C 0 sunt coliniare.Analog se procedeaz pentru O punct impropriu.
(q.e.d.).

73

Articole i note matematice

C0

B
C

Fig.15

B0
A
A0

B
b

Suficiena.Fie A0 , B0 , C 0 coliniare pe .Pe fasciculul de vrf B0 i raze , B0 C , B0 A


considerm perechile de puncte : B0 , C 0 ; C , A B0 C ; C A B0 A .Deoarece
triunghiurile A0 CC i C 0 AA sunt omologice, din teorema Desargues (direct)
rezult :
A0 C C 0 A = B, CC AA = O i C A0 AC 0 = B sunt coliniare; O BB .(q.e.d.).
Teorema II.12. Teorema lui Desargues direct cazul spaial.
Fie Oabc fasciculul de vrf O - propriu (fig.16)sau Oi - impropriu(fig.17) i
A, A a; B, B b; C , C c .n aceste condiii, punctele proprii sau improprii
A0 = BC B C , B0 = CA C A, C 0 = AB AB sunt coliniare pe dreapta d
(proprie ) sau d i (improprie).

74

Articole i note matematice

Demonstraie.

Fig. 16

A B

O punct propriu

C
A

C
A0

d
C0

B0

Oi
A
Fig. 17
Oi punct impropriu

C
C

d
A0

C0

B0
a

Distingem urmtoarele plane determinate de triplete de puncte necoliniare i perechi de


drepte concurente sau paralele: ( ABC ), ( AB C ), (a, b), (b, c) i (c, a) .
75

Articole i note matematice

Avem: ( ABC ) ( AB C ) = d , (bc) d = A0 d , (ca) d = B0 d , (ab) d = C 0 d ,


rezult A0 , B0 , C 0 coliniare. (q.e.d.).

Dac axa omologiei este dreapta improprie d i , rezult AB AB , BC B C , CA C A , iar


dac

vrful O este
CA
AB
BC
.
k=
=
=
AB B C C A

propriu

avem

omotetie

de

centru

modul

Teorema II.13. Teorema lui Desargues reciproc cazul spaial.


Fie A0 = BC B C , B0 = CA C A, C 0 = AB AB coliniare pe dreapta d (proprie
) sau d i (improprie).Atunci AA BB CC = O .
Demonstraie.Considerm fasciculul de vrf B0 ale crui raze sunt determinate de
tripletele de puncte coliniare ( B0 , C 0 , A0 ); ( B0 , C , A); ( B0 , C , A) i triunghiurile
omologice A0 CC , C 0 AA .n continuare se aplic teorema 12 i rezult c
B = A0 C C 0 A; B = A0 C C 0 A
i
O = CC AA
sunt coliniare, deci
AA BB CC = O .(q.e.d.).
Observaie. i n cazul plan (teorema II.11.), omologia de centru O - propriu i ax
improprie este o omotetie, iar omologia de centru impropriu i ax improprie este o
translaie.
Demonstraie.I). Dac a b c = O i d i - dreapt improprie, rezult

AB AB , BC B C , CA C A ,

deci

OAB OAB , OBC OB C

OA OB OC
=
=
= k , de unde A, B , C sunt
OA OB OC
omoteticile punctelor A, B, C n raport cu centrul O de modul k .(q.e.d.).
OCA OC A .De aici avem

II). Dac a b c i AB AB , BC B C , CA C A , rezult ABB A, BCC B , CAAC sunt


paralelograme.De aici AA = BB = CC = t , deci avem o translaie a ABC n direcia
comun a b c pe distana t .(q.e.d.).
Teorema II.14.Teorema lui Pascal pentru hexagon.
ntr-un hexagon nscris ntr-un cerc, laturile opuse se taie n puncte coliniare.
Demonstraie.Pentru claritatea desenului vom considera hexagonal stelat AB CABC
nscris n cercul C (O; R) cu perechile de laturi opuse ( AB , AB); ( B C , BC ) ; (CA, C A)
(fig.18.).Fasciculele de vrfuri A i C ale cror raze trec prin A, B , B, C sunt egale
(conform teoremei II.3.).
(1) A( BAB C ) = C ( BAB C ) .
Fie X = BA AC i tiem fasciculul de vrf A cu secanta BA , apoi din teorema
II.2, rezult :
(2) A( BAB C ) = C ( BAB C ) .
Fie Y = BC AC i tiem fasciculul de vrf C cu secanta BC .Din teorema II.2,
rezult :

76

Articole i note matematice

Fig.18
B
A
X
W

Y
V

U
C

A
B

(3) C ( BAB C ) = ( BYUC ) .


Din relaiile (1), (2) i (3), rezult (4) ( BAWX ) = ( BYUC ) = diviziuni anarmonice
egale cu B punct comun..
Din teorema II. 7., i schema de mai jos:
Puncte corespondente
B
B
comun
A
Y
dreapta AY
W
U
dreapta WU
C
X
dreapta XC
rezult AY ,WU , XC
sunt concurente.Deoarece, AY XC = V , avem
V WU .(q.e.d.).
Teorema II.15.Teorema lui Pascal pentru patrulatere nscrise.
ntr-un patrulater inscriptibil, inerseciile laturilor opuse i cele ale tangentelor la cercul
circumscris duse prin perechile de vrfuri opuse sunt 4 puncte coliniare.
Demonstraie.
Fie U = AB CD, V = BC AD , iar a i c tangentele n A i C la cercul
C (O; R) (fig.19).
Considerm W = a c .
Din teorema 5,
A( BaDC ) = C ( BADC ) , dar din teoremele I.7. i I.8. avem
A( BaDC ) = A(CDaB) ,
C ( BADc) = C ( ABcD) .Rezult
A(CDaB) = C ( ABcD) ,
fascicule anarmonice egale cu raza comun AC .

77

Articole i note matematice

Fig.19

a
A

C
c

Raze corespondente:
AC
CA
raz comun
CB
AD CB = V
AD
a
c
ac =W
AB
CD
AB CD = U
Rezult c: (1) U ,V , W sunt coliniare.
Raionnd analog pentru tangentele b i d , duse prin punctele B i D de pe C (O; R) ,
cu T = b d , deducem (2) T UV .Din (1) i (2) rezult c U , V ,W , T sunt
coliniare.(q.e.d.).
Teorema II.16.Teorema lui Brianchon.
ntr-un hexagon circumscris unui cerc diagonalele sunt concurente.
Demonstraie.Fie ABCDEF un hexagon circumscris cercului C (O; R) .Considerm
tangentele a, c, e i f tiate, pe rand, de tangentele b i d , unde
G = b f , H = b e, I = d f i J = d a (fig.20).Aplicnd teorema II.4, rezult
( BCHG) = ( JDEI ) , i permutnd convenabil (vezi partea I) obinem
(GHCB) = ( IEDJ ) .
n continuare, considerm fasciculele anarmonice cu vrfurile n F i A :
F (GHCB) = A( IEDJ ) , egale i cu raza comun FG = AI = f .

78

Articole i note matematice

Fig. 20

f
E

D
b

Conform teoremei II.8., i schema de mai jos


Raze corespondente:
AI
FG
raz comun
FH
AE
FH AE = E
FC
AD
FC AD = P
FB
AJ
FB AJ = B
rezult P EB , adic diagonalele AD, FC i EB sunt concurente n P .(q.e.d.).
Teorema II.17.Teorema lui Newton.
ntr-un patrulater circumscriptibil, cele dou diagonale mpreun cu cele dou drepte
determinate de punctele de contact ale perechilor de laturi opuse cu cercul nscris, sunt
patru drepte concurente.
Demonstraie.Fie ABCD patrulaterul circumscris cercului C (O; R) ; E , F , G, H
AB, BC , CD, DA
cu
cercul
punctele
de
contact
ale
laturilor
C (O; R) ; I = AB CD, J = BC AD (fig.21).Aplicm teorema II.6. tangentelor n H
i F la cerc, avem: ( HAJD) = ( JBFC ) , i permutnd convenabil ( HAJD) = ( JDHA) ,
deci : ( JDHA) = ( JBFC ) , diviziuni anarmonice egale, cu un punct comun J .Conform
teoremei II.7. i schemei de mai jos:

79

Articole i note matematice

J
D

H
A

G
C

E
F
Fig.21

Puncte corespondente
J
J
punct comun
D
B
dreapta DB
H
F
dreapta HF
A
C
dreapta AC
Q = AC BD , rezult (1) Q HF .
Raionm analog pentru diviziunile de pe tangentele n E i G la cerc, i obinem
( IBEA) = ( IDGC ) , apoi cu schema de mai jos:
Puncte corespondente
I
I
punct comun
B
D
dreapta BD
E
G
dreapta EG
A
C
dreapta AC
Cum AC BD = Q , rezult (2) Q EG .
Din (1) i (2) rezult AC , BD, EG, i FH sunt concurente n punctual Q .
Teorema II.18.ntr-un trapez isoscel, intersecia laturilor neparalele i interseciile
tangentelor duse prin vrfurile opuse la cercul circumscris trapezului, sunt puncte
coliniare pe o dreapt paralel cu bazele.
Demonstraie.Fie ABCD ,un trapez isoscel cu AB CD , i AD = BC .
Considerm a, b, c i d tangentele la cercul C (O; R) circumscris trapezului,duse prin
vrfurile trapezului.Notm: U = AD BC ,W = a c, T = d b i AB CD = Vi
(punctul impropriu pe direcia paralelelor AB i CD , Vi AB i Vi CD ) vezi
fig.22.

80

Articole i note matematice

W
D

c
Fig.22

teorema
II.5.,
avem
irul
de
Din
egaliti: A(aBCD) = B( AbCD) = C ( ABcD) = D( ABCd ) , care grupate cte dou i
permutnd convenabil A(aBCD) = C ( ABcD) , rezult A(CDaB) = C ( ABcD) , fascicule
anarmonice egale cu raza AC comun.Conform teoremei II.8., din schema de mai jos:
raze corespondente:
AC
CA
raz comun
AD
CB
AD CB = U
a
c
ac =W
CD
AB
AB CD = Vi
deducem Vi UW .Rezult (1) UW AB CD .
n
continuare
se
procedeaz
ca
mai
sus,
B( AbCD) = D( ABCd ) B( DCbA) = D( BAdC ) , fascicule anarmonice egale cu raza
BD comun.
Raze corespondente:
BD
DB
raz comun
BC
BC DA = U
DA
b
d
bd =T
BA
DC
BA DC = Vi
rezult Vi UT , deci (2) UT AB CD .
Din (1) i (2) rezult c punctele W , U i T sunt situate pe o dreapt paralel cu AB i
CD .(q.e.d.).
Teorema II.19.Raportul anarmonic al unui fascicul este egal cu raportul anarmonic al
coeficienilor unghiulari ai razelor fasciculului.
Demonstraie. Fie O(abcd ) fasciculul de vrf O i raze a, b, c i d .Considerm o

dreapt OX , astfel nct : = xa, = xb, = xc, = xd .

81

Articole i note matematice

p
d

D
C

A
X

a
x
Fig.23

p Ox
i
notm
Ducem
o
dreapt
: X = p Ox, A = p a, B = p b, C = p c, D = p d (vezi fig.23).
CA DA XA XC XA XD
:
=
:
, ns notnd
Rezult r = O(abcd ) = ( ABCD) =
CB DB XB XC XB XD
OX = x, tg = ma , tg = mb , tg = mc , tg = md ,
avem
m ma md ma
XA = xma , XB = xmb , XC = xmc , XD = xmd .Deci r = c
:
.(q.e.d.).
m c mb m d mb
Aplicaie.Polara unghiular.
Fie ( xy) , un unghi de vrf O i laturi x, y , iar A un punct nesituat pe laturile lui.O
secant mobil care conine punctul A taie laturile unghiului n punctele X i Y .
Se cere locul geometric al punctului M , conjugatul armonic al lui A n raport cu X
i Y .
Soluie.Se aplic teorema II.19., pentru :
not

not

not

a = x, b = OM , c = y, d = OA, ma = 0, mb = m, mc = m0 , md = = m A , i r = 1 .
m0
mA
2
1
1
+
=0 =
+
.
m0 m m A m
m m0 m A
Deci m = cons tan t. Locul geometric este OM , de direcie fix, cu coeficientul
unghiular fa de OX egal cu media armonic ai coeficienilor unghiulari ai dreptelor
OY i OA fa de OX .

Rezult :

Justificarea motto-ului ales este :


Pentru construcia geometriei proiective se utilizeaz metoda axiomatic
sintetic.Dup ce se construiete corpul coordonatelor asociat unui spaiu proiectiv sau
plan proiectiv desarguesian, se poate trece, n cazul unui corp comutativ, la dezvoltarea
geometriei analitice (n coordonate ) proiective.Geometria afin se recupereaz pe
82

Articole i note matematice

complementara unui hiperplan al spaiului proiectiv;se poate face deci trecerea de la


proprieti proiective la proprieti afine i reciproc.

XIII. Solving problems of concurrence and collinearity


using properties of pencils of lines
Projective geometry is whole geometry
Arthur Cayley

Neculai N. Stanciu
Abstract. This article is devoted to the study of two fundamental and reciprocal
questions: when do three given points lie on a single line, and when do three given lines
pass through a single point? The techniques we describe in this article will be
augmented by more sophisticated approaches, such as the Papuss theorems, the
Desarguess theorems, the Pascals theorem and the Brianchons theorem.
The formalism of projective geometry makes a discussion of such properties
possible, and exposes some remarkable facts, such as the duality of points and
lines.While technique cross-ratio of four points, and in the light of duality the crossratio of four lines can be useful on contest problems, much of the material here is
considered too advanced for primary and secondary school education.This is a pity, as
some of the most beautiful classical geometry appears only in the projective geometry.
Key words: cross-ratio, bivalent range, harmonic range, harmonic conjugate,
concurrence and collinearity .
AMS Classification. 51-xx,51Axx, 51A05.

1. Main purpose - of the results below is familiarizing readers with new


methods ( little known even teachers of mathematics) solving problems of concurrence
and collinearity namely the techniques offered by pencils of lines properties.
We consider fig.1 where S (a, b, c, d ) or S ( A, B, C , D) represents a convergent pencil of
lines, with its own point S and rays a, b, c, d or SA , SB, SC , SD and fig.2
where S (a, b, c, d ) is a parallel pencil of lines with rays a, b, c, d or SA , SB, SC , SD ( S
is improperly point).
S
Fig.1
Fig.2

83

Articole i note matematice

CA DA
:
is harmonic ( ( ABCD ) = 1 ) then the pencil
CB DB
attachment S ( ABCD) is called harmonic pencil of lines.
def

If the cross-ratio ( ABCD) =

2. Cross-ratio corresponding to a convergent pencil of lines


We consider the pencil of lines S (abcd ) cut by line (you see fig.1) in the points
not

A = a, B = b, C = c, D = d .If S ( XYZ ) = triangle area with vertices


not

CA CA h 2 S (CSA) S (CSA)
X , Y and Z , XY = XSY , h = d ( S , ) , then
=
=
.
=
CB CB h 2 S (CSB ) S (CSB )

CA DA S (CSA) S ( DSA) SC SA sin(ca) SD SA sin( da)


=
=
=
( ABCD) =
:
:
:

CB DB S (CSB) S ( DSB) SC SB sin(cb


) SD SB sin(db)

sin(ca) sin(da)
=
.
:

sin(cb) sin(db)

sin(ca) sin(da)
If S (abcd ) =
, then results ( ABCD) = S (abcd ) .
:

sin(cb) sin(db)
not

3. Properties (invariants theorems)


Theorem 1.On a line we consider four fixed points A, B, C , D .For any S , we
denoted a = SA, b = SB, c = SC , d = SD . Cross-ratio corresponding to a convergent
pencil of S (abcd ) is invariant.
Proof.Let S , S , so fig.3.
S
Fig.3
S

A
a

84

Articole i note matematice


a

Because S (abcd ) = ( ABCD) and S (a bc d ) = ( ABCD) results


S (abcd ) = S (a bc d ) .(q.e.d).
Theorem 2.We consider fixed pencil of lines with vertix S and rays a, b, c, d .For any
secant line which intersect the rays of pencil in A = a , B = b , C = c , and
D = d , Double-ratio corresponding to division ( ABCD) este invariant.
Proof.Let and two some secant lines (you see fig.4), which intersect the rays of
the pencil of lines in the points A, B, C , D and A, B , C , D .

Fig.4

S
A

B C D

We have ( ABCD) = S (abcd ) and ( AB C D ) = S (abcd ) .Hence


( ABCD) = ( AB C D ) .(q.e.d.).
Pencil of lines cut by a secant paralell with one of the rays.
Let S (abcd ) be a pencil of lines and a (you see fig.5).

Fig.5

S
C

B
a

C A D A
C A SA
,(2) C AS C BC
,
:
=
C B D B
C B CB
D A SA
(3) D AS D BD
=
.Under (1),(2) and (3) results :
D B DB
SA SA
1
1
S (abcd ) =
:
=
:
= ( ABC D ) .
CB DB CB DB

(1) S (abcd ) = ( ABC D ) =

85

Articole i note matematice

1
1
:
.
CB DB
So a scriem a = Ai (improperly point on the direction parallels a ),
We have the following mnemotehnical rule for writing the double-ratio

S (abcd ) =

CAi DAi
:
and we take CAi : DAi = 1 (switching to limit A Ai ).
CB DB

1
1
:
.
CB DB
Corollary.Let B, C , D be the fixed points on a line ,
a , S a , SB = b, SC = c, SD = d .

S (abcd ) =

Then S a, S (abcd ) is invariant.


Proof.Let Ai = a . S (abcd ) = ( Ai BCD ) =

CAi DAi
1
1
:
=
:
=constant.
CB DB CB DB

Theorem 3.Let A, B, C , D be the fixed points on C (O; R) and M C (O; R) (you see
fig.6).If MA = a, MB = b, MC = c, MD = d then , M C (O; R) M (abcd ) is invariant.

sin(ca) sin(da )
Proof. M (abcd ) =
=constant, because A, B, C , D are fixed points and
:

sin(cb) sin(db)

CB
DCBA
DCB
CBA
=ct., cb =
=ct., da =
=ct., db =
=ct.
ca =
2R
2R
2R
2R

Observation. You see figure 6, results M ( ABCD) = M ( AB C D ) .


Theorem 4. Let A, B, C , D be fixed points on C (O; R) and , a, b, c, d the tangents in
the four points at circle C (O; R) .Then whatever tangent t to the circle C (O; R) in point
T C (O; R) , the points A1 = a t , B1 = b t , C1 = c t i D1 = d t formed a
invariant division ( A1 B1C1 D1 ) .
Proof. We have figure 7

86

Articole i note matematice

Fig.7
( A1 B1C1 D1 ) = O( A1 B1C1 D1 ) .We consider the pencil of lines with vertix T and the rays
TA OA1 , TB OB1 , TC OC1 , TD OD1 .So T ( ABCD ) = O( A1 B1C1 D1 ) .

CBA
CB
DA
BCD
We get ( A1 B1C1 D1 ) = T ( ABCD ) = (sin
: sin
) : (sin
: sin
) =constant.
2R
2R
2R
2R
Theorem 5.On circle C (O; R) consider distinct points A, B, C , D and tangent a, b, c, d
in these points at the circle (you see fig.8).We have:
A(aBCD) = B( AbCD) = C ( ABcD) = D( ABCd ) .

Proof. A(aBCD) = (sin CAa : sin CAB) : (sin DAa : sin DAB) =

CDA
BC
DA
BCD
: sin
) : (sin
: sin
) = r = constant=
= (sin
2R
2R
2R
2R
= B( AbCD) = C ( ABcD) = D( ABCd ) (from the equalities of sines).

Fig. 8
Observation. Theorem 5 represents the limit case of the theorem 3 the point M on
the C (O; R) is one of the points A, B, C or D .
Teorema 6.On circle C (O; R) we consider the distinct points A, B, C , D and the
tangents at the circle in these points a, b, c, d (you see fig.9).
If denoted E = a b, F = b c, G = c d , H = d a, I = b d and J = a c, then
we have the equalities : ( AEJH ) = ( EBFI ) = ( JFCG) = ( HIGD) .

87

Articole i note matematice


Proof.We consider the pencil of lines with vertex O and rays OA, OE , OJ , OH

J
Fig.9
OAEJH , then the pencil of lines with vertex in A and the rays perpendiculars on the
rays of previously pencil of lines : a OA, AB OE , AC OJ , AD OH ,
A(aBCD) and we have
( AEJH ) = O( AEJH ) = A(aBCD) .
The same is obtained the equalities:
( EBFI ) = O( EBFI ) = B ( AbCD) ;
( JFCG ) = O( JFCG ) = C ( ABcD) ;
( HIGD) = O( HIGD) = D( ABCd ) .
Now we use the equalities:

CDA
ABC
CDA
CAa = CBA = cCA =
i CDA =
=
, and results:
2R
2R
2R

CDA
sin(CAa ) = sin(CBA) = sin(cCA) = sin(CDA) = sin(
) .
2R
The same is obtained the equalities:

CB
sin(CAB ) = sin(CBb) = sin(cCB) = sin(CDB ) = sin( )
2R

DA
sin( DAa) = sin( DBA) = sin( DCA) = sin( dDA) = sin(
)
2R

DAB
sin( DAB) = sin( DBb) = sin( DCB ) = sin( dDB) = sin(
).
2R
Given these values can write:

CDA
CB
DA
DAB
A(aBCD ) = B( AbCD ) = C ( ABcD) = D( ABCd ) = (sin
: sin
) : (sin
: sin
)
2R
2R
2R
2R

(q.e.d.).
Observation. Theorem 6 represents the limit case of theorem 4 - the tangenta t at the
circle C (O; R) is one of the tangents a, b, c, d .

4. Theorems on concurrence and collinearty


88

Articole i note matematice


Theorem 7. If ( ABCD) = ( AB C D ) - have a common point A , then the lines
BB , CC , DD are concurrence.
Proof.We have the fig.10.
O

Fig.10

B
A

Let O = BB CC and D = OD .We use the theorem 2 and results


( ABCD) = ( AB C D ) , now use the hypothesis and we have ( ABCD) = ( AB C D ) .
Hence ( AB C D ) = ( AB C D ) , then D = D .(q.e.d.).
Theorem 8.If S (abcd ) = S (abc d ) - common ray SS = a , then the points of
intersection of the three pairs of rays correspondent: B = b b, C = c c , D = d d
are collinear.
S
Fig.11

c
B

S
Proof.
Let A = BC a, D = BC d , D = DC d (fig.11).
From the hypothesis we get (1) S (abcd ) = S (abc d ) .We intersect the pencil of lines
S (abcd ) with BC and results (2) S (abcd ) = ( ABCD) .We intersect the pencil of lines
S (ab c d ) with BC and results (3) S (abc d ) = ( ABCD ) .From this three relations we
get ( ABCD) = ( ABCD ) , then D = D .(q.e.d.).

5. At the end - I propose some classical theorems that can be attacked with pencils
of lines techniques(theorem 7 and theorem 8).
Teorema 9. Pappuss theorem
89

Articole i note matematice

If A, B and C are three points on one line , D, E and F are three points on another
line , and AE meets BD at X , AF meets CD at Y , and BF meets CE at Z , then the
three points X , Y and Z are collinear.
Teorema 10.Desarguess theorem.
In a projective space, two triangles are in perspective axially if and only if they are in
perspective centrally.
To understand this, denote the three vertices of one triangle by (lower-case) a, b, and c,
and those of the other by (capital) A, B, and C. Axial perspectivity is the condition
satisfied if and only if the point of intersection of ab with AB, and that of intersection of
ac with AC, and that of intersection of bc with BC, are collinear, on a line called the axis
of perspectivity. Central perspectivity is the condition satisfied if and only if the three
lines Aa, Bb, and Cc are concurrent, at a point called the center of perspectivity.
Theorem 11. Pascals theorem (The dual of Brianchon's theorem ).
Given a (not necessarily regular, or even convex) hexagon inscribed in a conic section,
the three pairs of the continuations of opposite sides meet on a straight line, called the
Pascal line.
Theorem 12. Brianchons theorem (The dual of Pascals theorem).
Given a hexagon circumscribed on a conic section, the lines joining opposite polygon
vertices (polygon diagonals) meet in a single point.

References

[1] http://www.nct.anth.org.uk/
[2] http://en.wikipedia.org/wiki/Projective_geometry
[3] http://robotics.stanford.edu/~birch/projective/
[4] http://www.math.poly.edu/courses/projective_geometry/
[6] http://www.cs.elte.hu/geometry/csikos/proj/proj.html
[7] http://www.geometer.org/mathcircles/projective.pdf
Neculai Stanciu
Department of mathematics
High School Saint Mc. Sava Berca
Micro V, Bl. 36, Ap. 15, Buzu, Romania
stanciuneculai@yahoo.com

90

Articole i note matematice


Bibliografie

[1] Nicolescu, L., Boskoff, W., Probleme practice de geometrie, Ed. Tehnic, Bucureti,
1990.
[2] Mihileanu, N. N., Complemente de geometrie sintetic, E.D.P., Bucureti, 1965.

Neculai STANCIU (CV)

Profesor de matematic
Studii
Master n management educaional i comunicare instituional, SNSPA, Bucureti
(2005 2007)
Liceniat n matematic, Universitatea Bucureti (1992 1998)
Liceniat al Facultii de Tehnologia Construciilor de Maini, Universitatea Tehnic
Gh. Asachi Iai (1987 1992)
Cursuri Postuniversitare pentru profesionalizarea pedagogic a absolvenilor de
nvmnt superior, Universitatea Politehnic Bucureti, (1994 )
Experien profesional
Titular la c.GeorgeEmil Palade i c. Nr.6 Buzu (din 2009)
Titular la Grupul colar Tehnic Sf. Mucenic Sava , Berca, Buzu (1997 2009)
Lider de sindicat la Grupul colar Tehnic Sf. Mucenic Sava , Berca, Buzu (2004
2005)
Director adjunct la Grupul colar Tehnic Sf. Mucenic Sava , Berca, Buzu (2005)
Director la Grupul colar Tehnic Sf. Mucenic Sava , Berca, Buzu (din 2006)
Lider de sindicat la Grupul colar Tehnic Sf. Mucenic Sava , Berca, Buzu (2008)
eful catedrei de matematic (2008 2009)
Titular la coala George Emil Palade Buzu i coala nr. 6 Buzu (2009 2010)
Lucrri publicate
Reflecii Metodice i Psihopedagogice, Editura Casa Corpului Didactic I. Gh.
Dumitracu, Buzu, 2005 (coautor)
Matematic de vacan, Editura Rafet, Rm. Srat, 2006 (coautor)
Monografie Zona Berca Buzu, Editura Casa Corpului Didactic I. Gh.
Dumitracu, Buzu, 2006 (coautor)
Matematic gimnaziu & liceu, Editura Rafet, Rm. Srat, 2007
Elemente de Management Educaional, Editura Rafet, Rm. Srat, 2007
(coautor)
Peste 100 de probleme i comunicri tiinifice
Peste 30 de articole de pedagogie i metodic
Altele
Membru n Biroul de conducere al Societii de tiine Matematice din Romnia, filiala
Rmnicu Srat
Directorul revistei de cultur matematic pentu tineret Sclipirea Minii

91

Articole i note matematice

CUPRINS

I.

Prefa /
Istoricul noiunilor matematice studiate n gimnaziu i liceu /
Bibliografie /

II.

Probleme rezolvate /
Bibliografie /

III.

Inegalitatea izoperimetric /
Bibliografie /

IV.

Aplicaii ale coordonatelor baricentrice /


Bibliografie /

V.

Teoreme fundamentale ale algebrei liniare, geometriei afine i euclidiene /


Bibliografie /
VI.
Calculul integral pentru funciile pare i impare generalizate /
Bibliografie /
VII. Calculul integral n cazul funciilor periodice /
Bibliografie /
VIII.
IX.
X.
XI.
XII.
XIII.

Rezolvarea analitic i sintetic a unor probleme de geometrie n spaiu /


Asupra unei propoziii i aplicaiile ei /
Generalizarea unor inegaliti /
Despre irul lui Fibonacci /
Diviziuni i fascicule anarmonice /
Solving problems of concurrence and collinearity using properties of
pencils of lines /

Moto:
Contiina datoriei mplinite prelungete viaa

92

You might also like